You are on page 1of 39

NEET 2023-24

2
Chapter

Motion in a
Straight Line
FOR BUY WHATSAPP https://wa.me//+919583963673

T1: Rest & Mo tio n, Mo tio n in One,


Two, Three Dimensions, Frame of
Reference, Position, Path Length,
Displacement, Speed and Velocity

T2: Acceleration, Kinematic Equations


for Uniformly Accelerated Motion

T3: Motion Under Gravity

T4: Relative Velocity in One Dimension


32 Physics

9583963673
Motion in a Straight Line 33

Rest & Motion, Motion in One, Two, Three Dimensions, Frame of Reference,
TOPIC 2.1 Position, Path Length, Displacement, Speed and Velocity

INTRODUCTION Rest and motion are relative terms :


Ø Mechanics is the branch of physics that deals with the Ø When we say that an object is at rest or in motion, then
conditions of rest or motion of material objects. this statement is incomplete and meaningless. Basically,
Mechanics is divided into two branches: rest and motion are relative terms. An object which is at
(a) Statics : It deals with the law of combination of forces rest can also be in motion simultaneously. This can be
and the conditions of equilibrium of solid, liquid and illustrated as follows.
The passengers sitting in a moving bus are at rest with
gaseous states of the bodies.
respect to fellow passangers but they are also in motion
(b) Dynamics : It includes
at the same time with respect to the objects like trees,
(i) Kinematics : It deals with the study of
buildings on the road side etc. So the motion and rest are
mechanical motion of bodies without taking into
relative terms.
account the forces which cause the motion of
the bodies. MOTION IN ONE, TWO AND THREE
(ii) Kinetics : It deals with the study of mechanical DIMENSIONS
motion of bodies by taking into account the Motion in One Dimension
forces which cause the motion. Ø An object moving along a straight line or path is said to
Ø Point object : An object is said to be a point object, if its have one-dimensional motion, also known as rectilinear
motion.
dimensions (i.e. length, breadth, thickness etc.) are
Ø Suppose, an object moves in a particular direction then
negligible in comparison to the distance travelled by it.
its position at any time can be described by knowing its
For example, an aeroplane, which flies from Delhi to distance from the starting point of the motion. It means,
London, a train moving over long distances etc. All these only one position coordinate (say, x) is required to describe
objects i.e. an aeroplane, a train etc. are treated as point the position of the object in one dimensional motion.
objects as their size or dimensions are negligible in Starting point Position after time t Position after time t'
comparison to the distance travelled by them.
O A B X
REST AND MOTION
x
Ø Rest : When position of a body does not change with
x'
time it is said to be in state of rest.
Ø Examples : Motion of a bus on a straight road and motion
Ø Motion : When position of a body changes with time it is of a train on a straight track, an object dropped from a
said to be in state of motion. certain height above the ground, etc.
(i) Absolute motion : The motion of a body with respect Motion in Two Dimensions
to a body which is at complete rest is called absolute Ø An object moving in a plane is said to have two
motion. (which is impossible). dimensional motion. When an object moves in a plane,
(ii) Relative motion : Comparing the motion of two its position can be described by knowing the co-ordinates
objects relative to each other. The motion of a body in two mutually perpendicular coordinate axes (say x and y).
with respect to a body which is at rest relative to the Since two coordinates (x, y) are required to describe the
body is called relative motion. position of the object, hence its motion is said to be two
For example, a person standing in moving bus is at dimensional.
rest with respect to bus driver but he is in motion Y
B(x2, y2)
with respect to a tree on road.
Ø If all the three co-ordinates x, y and z of the particle remain
(x1, y1) A
unchanged as time passes, we say that the particle is at
rest with respect to this frame. If any one or more
coordinates changes with time, we say that the body is y2
moving with respect to this frame. y1
Ø It should be noted that object position may change with
time due to change in only one or two or all the three co-
X
ordinates. O
x1
x2

9583963673
34 Physics

Ø Examples : Motion of an insect on a floor, earth revolving Example 1


around the sun, a billiard ball moving over the billiard
table, etc. A bird is sitting on a branch of tree whose co-ordinates are
(1, 2, 3). A hunter is standing on the ground. Represent the
Motion in Three Dimensions position of the bird with respect to the hunter.
Ø An object moving in space is said to have three Sol. ®
dimensional motion. In three dimensional motion, three Using : r = x iˆ + y ˆj + z kˆ
coordinates (x, y and z) are required to describe the ® ®
position of the object at any instant fig. r = (1) ˆi + (2) ˆj + (3) kˆ ; r = ˆi + 2 ˆj + 3kˆ

Y B(x2, y2, z2) Path Length or Distance


(x1, y1, z1)
Ø The length of the actual path between initial and final
A
positions of a particle in a given interval of time is called
distance covered by the particle. Distance is the actual
y1 y2 length of the path. It is the characteristic property of any
O path i.e. path is always associated when we consider
z2 z1 x1 X
distance between two positions.
Ø Distance between A and
B while moving through 2
Z path (1) may or may not
x2
be equal to the distance
Ø Examples : Motion of a kite, motion of a flying aeroplane between A and B while
or bird, etc. moving through path (2). A 1 B
FRAME OF REFERENCE Characteristics of distance
y (i) It is a scalar quantity
Ø To locate the position (ii) It depends on the path
of a particle we need a (iii) It never reduces with time.
reference frame. A (iv) Distance covered by a particle is always positive and
commonly used can never be negative or zero.
reference frame is (v) Dimension : [M0 LT0]
cartesian coordinate O x (vi) Unit : In C. G. S. centimetre (cm), in S.I. system,
system or simply metre (m).
coordinate system.
Displacement
z
Ø The coordinates (x, y and z) of a particle specify the position Ø The shortest distance from the initial position to the final
of the particle with respect to origin (O) serves as the position of the particle is called displacement. The
reference point of that frame. It is the point of intersection displacement of a particle is measured as the change in
of these three co-ordinates x, y and z. If all the three the position of the
coordinates of the particle remain unchanged as time passes particle in a particular
it means the particle is at rest w.r.t. this frame. The reference direction over a given Y
A
frame is chosen according to the problems. If the frame is time interval. It r
not mentioned, then ground is taken as the reference frame depends only on final rA B
For example, the three mutually perpendicular axes i.e. Ox, and initial positions. rB
Oy and Oz form a reference frame. Ø Displacement of a
X
particle is a position
POSITION, PATH LENGTH AND vector of its final O
DISPLACEMENT position w.r.t, initial
Position position.
Ø The position of any particle can be given as follows : Z
®
In cartesian co-ordinate form : The position of any particle Position vector of A w.r.t. O = OA
is represented by co-ordinates (x, y, z) or position vector ®
® Þ rA = x1 ˆi + y1 ˆj + z1 kˆ
( r ). If a particle is located at point A in frame of reference ®
Position vector of B w.r.t. O = OB
x, y, z then the position of particle will be ®
® Þ rB = x 2 ˆi + y 2 ˆj + z 2 kˆ
r = x ˆi + y ˆj + z kˆ Y Displacement
p(r, q) ®
In polar form : x = r cosq ; y = r sinq r = AB = (x 2 - x1 ) ˆi + (y2 - y1 ) ˆj + (z 2 - z1 ) kˆ
® q r
r = r cos q iˆ + r sin q ˆj O X Dr = Dx ˆi + Dy ˆj + Dz kˆ
Motion in a Straight Line 35

Characteristics of displacement ^j
(i) It is a vector quantity.
(ii) The displacement of a particle between any two
points is equal to the shortest distance between them. –OA B
C
(iii) The displacement of an object in a given time interval
may be +ve, –ve or zero.

OB
(iv) The actual distance travelled by a particle in the given OB

–O
interval of time is always equal to or greater than the ^i

A
magnitude of the displacement and in no case, it is
A
less than the magnitude of the displacement, i.e.
Distance ³ | Displacement | O
(v) Dimension : [M0 L T0] uuur uuur uuur
Sol. Displacement AB = OB - OA = 10jˆ - 10iˆ
(vi) Unit : In C. G. S. centimetre (cm), in S. system, uuur
metre (m). | AB | = 102 + 102 = 10 2m
OA 10
Comparative Study of Displacement and Distance From D OBC, tan q = = = 1 Þ q = 45°
OB 10 uuur
S. No. Displacement Distance Angle between displacement vector OC and x-axis
= 90° + 45° = 135°
It has single value It may have more than one 1 1
1
between two points. value between two points. Distance of path AB = (circumference) = (2pR) m = (5p) m
4 4
May be +ive, –ive or Example 6
2 It is always > 0 (+ive)
zero. A man has to go 50 m due north, 40 m due east and 20 m due
It can decrease with It can never decrease with south to reach a field from his house.
3 (a) What distance he has to walk to reach the field?
time. time.
(b) What is his displacement from his house to the field?
4 It is a vector quantity It is a scalar quantity.
Sol. N A 40m
B
Example 2
W E 20m
An old person moves on a semi circular track of radius 40 m
during a morning walk. If he starts at one end of the track and 50m C
S
reaches at the other end, find the displacement of the person.
Sol. Displacement = 2R = 2 × 40 = 80 m
O
D
Example 3 Let origin be O then :
(a) Distance covered
An athlete is running on a circular track of radius 50 m. = OA + AB + BC = 50 + 40 + 20 = 110 m
Calculate the displacement of the athlete after completing 5
rounds of the track. (b) First method : Displacement OC = OD 2 + CD 2
Sol. Since final and initial positions are same hence
= 402 + 302 = 50 m
displacement of athlete will be, Dr = r – r = 0
®
Example 4 Second method : Displacement d = 50 ˆj + 40 iˆ - 20 ˆj
= 30 ˆj + 40 ˆi
A monkey is moving on circular path of radius 80 m . Calculate
®
the distance covered by the monkey in one revolution. | d | = 402 + 302 = 50m
Sol. Distance = circumference of the circle
SPEED
d = 2 p R Þ d = 2 p × 80 = 160 × 3.14 = 502.40 m
Ø Speed of an object is defined as the rate of change of
Example 5 position of the object in any direction. It is measured by
the distance travelled by the object in unit time in any
A particle goes along a quadrant from A to B is a circle of radius direction.
10m as shown in figure. Find the direction and magnitude of
Distance travelled
displacement and distance along path AB. i.e., Speed =
Time taken
36 Physics

(i) It is a scalar quantity Ø If car covers equal distances with different speeds then,
(ii) It gives no idea about the direction of motion of the x1 = x2 = x3 = x
object.
3x 3 3v1 v 2 v3
(iii) It can be zero or positive but never be negative. V= = =
(iv) Unit : C.G..S. cm/sec, S.I. m/sec, x x x 1 1 1 v1v 2 + v 2 v 3 + v3 v1
+ + + +
(v) Dimensions : [ M0LT–1] v1 v 2 v3 v1 v 2 v3
Uniform Speed Case 2 : If any body travels with speeds v1, v2 and v3 during
Ø An object is said to be moving with a uniform speed, if it time intervals t1, t2 and t3 respectively then the average speed
covers equal distances in equal intervals of time, of the body will be
howsoever small these intervals may be. The uniform x + x 2 + x 3 v1t1 + v2 t 2 + v3 t 3
speed is shown by straight line in distance-time graph. Average speed V = 1 =
t1 + t 2 + t 3 t1 + t 2 + t 3
For example, suppose a train travels 1000 m in 60 sec. The
(v + v + v ) ´ t (v1 + v2 + v3 )
train is said to be moving with uniform speed, if it travels If t1 = t2 = t3 = t then V = 1 2 3 =
500 m in 30 sec, 250 m in 15 sec, 125 m in 7.5 sec and so on. 3´ t 3
Non-uniform Speed Instantaneous Speed
Ø An object is said to be moving with a non-uniform or Ø The speed of the body at any instant of time or at a
variable speed, if it covers equal distances in unequal particular position is called instantaneous speed. Let a
intervals of time or unequal distances in equal intervals of body travels a distance Dx in the time interval Dt, then its
time, howsoever small these intervals may be. Dx
average speed = .
For example, suppose a train travels first 1000 m in 60 sec, Dt
next 1000 m in 120 sec and next 1000 m in 50 sec, then the When Dt ® 0, then average speed of the body becomes
train is moving with variable speed or non-uniform speed. the instantaneous speed.
Average Speed Dx dx
\ Instantaneous speed = Lim =
Ø When an object is moving with a variable speed, then the Dt ® 0 Dt dt
average speed of the object is that constant speed with
which the object covers the same distance in a given time
as it does while moving with variable speed during the
given time. Average speed for the given motion is defined 1. Speedometer of the vehicle measures its instantaneous
as the ratio of the total distance travelled by the object to speed.
the total time taken 2. In case of a uniform motion of an object, the instantaneous
Total distance travelled speed and average speed is equal to its uniform speed.
i.e., Average speed V =
Total time taken
Example 7
The distance travelled by a particle in time t is given by x = 2.5
Ø If a particle travels with speed v1 for first half time of its t2 (m). Find the average speed of the particle during the time 0
total motion and with speed v2 for next half time then to 5 sec.
v1 + v2
vav = . Sol. Distance covered x = 2.5 t2
2 During time 0 to 5 sec.
Ø If any car covers distance x1, x2,......xn in the time x = 2.5 × (5)2 = 2.5 × 25 = 62.5 m
intervals t1, t2,....... tn then, average speed, Average speed
x1 + x 2 + x3 + ....... + x n x 62.5 62.5
V= V= = = = 12.5 m/s
t1 + t 2 + ......... + t n t 2 - t1 5 - 0 5

Some important cases related to average speed : Example 8


Case 1 : If a car covers distances x1, x2, and x3 with speeds v1, A train 150 m long is moving with a speed of 90 km/h. In what
v2, and v3 respectively in same direction then average speed of time shall it cross 850 m long bridge ?
car v1 v2 v3
Sol. Total distance to be covered = 850 + 150 = 1000 m
X' X 150
O x1 x2 x3
x1 + x 2 + x3 x x x
V= ; here, t1 = 1 , t 2 = 2 , t 3 = 3 850 m
t1 + t 2 + t 3 v1 v2 v3
x1 + x 2 + x 3
Þ V=
x1 x 2 x 3
+ +
v1 v 2 v 3
Motion in a Straight Line 37

5 If the speed varies with the time:


Speed = 90 km/h = 90 × m/s = 25 m/s ds
18 Then, v = Þ ds = v dt Þ òds = òv dt
dt
1000
Now, time = s = 40 s or s = òv dt
25
Example 12
Example 9
Find the distance travelled by the particle during the time t = 0
A bicyclist is travelling along a straight road for the first half to t = 3 sec from the figure.
time with speed v1 and for second half time with speed v2. What
is the average speed of the bicyclist? 6 B
Sol. Let t be the total time taken then distance covered in the
æ t ö v1t 4

V(m/s)
first half time = v1 çè ÷ø =
2 2 2
æ t ö v2 t
Distance covered in the next half time = v2 çè ÷ø = O
A
2 2 1 2 3 t(s)
v1 t v 2 t 1
+ Sol. Distance S = Area of DOAB = × OA × BA
Average speed vav. = 2 2 = v1 + v 2 2
t 2 1
= × 3 × 6 = 9 m.
Example 10 2

A person travels along a straight road due east for the first Example 13
half distance with speed v1 and the second half distance with If the speed of a particle is v = 10 t2 m/s. Find out distance
speed v2. What is the average speed of the person? covered from t = 2 sec to t = 5 sec.
Sol. Let S be the total distance travelled. 5 5

S/ 2 S Sol. s = ò v dt = ò 10 t²dt = 10 ò t²dt


Time taken for the first half distance = = 2 2
v1 2v1
( ) ( )
10 3 5 10 3 3
S/ 2 S = t = 5 - 2 ; 390 m.
= 3 2 3
Time taken for the second half distance =
v2 2v2
VELOCITY
S S
Total time taken = 2v + 2v Ø The rate of change of displacement of a particle with time
1 2 is called the velocity of the particle.
S 2v1v 2
Average speed, vav. = = Displacement
S S v1 + v 2 i.e., Velocity =
+ Time interval
2v1 2v 2
(i) It is a vector quantity
Example 11 (ii) The velocity of an object can be positive, zero and
negative
The distance travelled by a particle x = 10t2 m. Find the value of
instantaneous speed at t = 2 sec. (iii) Unit : C.G.S. cm/s, S.I. m/s.
(iv) Dimensions : [M0LT–1]
dx d
Sol. v = = (10t 2 ) = 10(2t) = 20 t
dt dt Uniform Velocity
Put t = 2 sec. Ø A body is said to move with uniform velocity, if it covers
v = 20 × 2 = 40 m/s. equal displacements in equal intervals of time, howsoever,
small these intervals may be.
Distance by speed-time graph : When the particle moves from
Ø When a body is moving with uniform velocity, then the
time t1 to t2 with uniform speed V as shown in the graph:
magnitude and direction of the velocity of the body
C
remains same at all points of its path.
B
Speed

Displacement

Velocity

A D
O t1 t2 Time
Then distance covered
S = V(t2 – t1) = AB × AD = Area of ABCD
Total distance travelled by particle = area of speed-time graph O Time O Time
38 Physics

Non-uniform Velocity From fig., the average velocity between points A and B is
Ø The particle is said to move with non-uniform or variable ® ® ®
® x 2 - x1 Dx
velocity, if it covers unequal displacements in equal V av = =
intervals of time, howsoever, small these time intervals t 2 - t1 Dt
may be. In this type of motion velocity does not remain Ø If time interval is small i.e. t2 – t1 = Dt
r
constant. The displacement-time graph, if a body moving ® ® ® ur Dx
with non-uniform speed is as shown in figure below x
and 2 1 - x = D x V
, then av = = tan q from graph (A)
Dt
Ø Average velocity is equal to slope of straight line joining
two points on displacement time graph. If Dt ® 0, then
Displacement

average velocity becomes instantaneous velocity.

x2 B

O Time Point P
A a C
Average Velocity x1
Ø The average velocity of an object is equal to the ratio of Dt
the displacement to the time interval for which the motion t1 t2
takes place Time
Displacement Graph (B)
i.e., Average velocity = ® ®
Time taken ®Dx d x
Ø Instantaneous velocity, V = Dt Lt
® 0 Dt
=
dt
y A or inst. velocity at point P
Displacement

®
r1 V = tan a (slope of tangent at point P)
B NEET 2016, 2022

r2 Comparative Study of Speed and Velocity


x
O t1 t2
Time S. No. Speed Velocity
r r 1 It is the rate of change It is the rate of change of
Ø If the initial and final position of a particle are r1 and r2 at
of position of a body. displacement of a
time t1 and t2 respectively,
particle.
® ® ®
Then displacement D r = r2 - r1 2 It tells nothing about It tells about the
and elapsed time Dt = t2 – t1 the direction of motion direction of motion of
® ® ®
of the particle. the particle.
r -r Dr ®
\ Average velocity V av = 2 1 = 3 It can be positive or It can be positive,
t2 - t1 Dt zero. negative or zero.
Instantaneous Velocity 4 It is a scalar quantity. It is a vector quantity.
Ø The velocity of the object at a given instant of time or at a
given position during motion is called instantaneous Example 14
velocity. A table clock has its minute hand 4 cm long. Find average
velocity of the tip of the minute hand (a) in between 6 a.m. to
6.30 a.m. and (b) 6 a.m. to 6.30 p.m.
x2 B Sol.
(a) At 6.00 a.m. the tip of the minute hand is at 12 mark and at
A C 6.30 a.m. or 6.30 p.m. it is 180º away. Thus the displacement
x1 between the initial and final positions of the tip is equal to
Dt the diameter of the clock.
t1 t2 Displacement = 2 R = 2 × 4 cm = 8 cm
Time Time taken from 6 a.m. to 6.30 a.m. is 30 minutes = 1800 s.
Graph (A)
Motion in a Straight Line 39

Displacement Example 16
The average velocity is Vav =
Time
The position of a particle moving on x-axis is given by
8
= = 4.4 × 10–3 cm/s x = At3 + Bt2 + Ct + D. The numerical value of A, B, C and D are
1800 1, 4, –2 and 5 respectively and S.I. units are used. Find velocity
(b) Again time taken from 6 am to 6.30 p.m. of the particle at t = 4 sec.
= 12 hrs + 30 minutes = 45000 s dx d
Displacement 8 Sol. V = = [At 3 + Bt 2 + Ct + D]
\ Vav = = = 1.8 × 10–4 cm/s dt dt
Time 45000 or V = 3At2 + 2Bt + C
Example 15 At time t = 4 sec.
A man walks on a straight road from his home to a market Considering A = 1, B = 4, C = –2
2.5 km away with a speed of 5 km/h. Finding the market closed, V = 3 A(4)2 + 2B(4) + C
he instantly turns and walks back with a speed of 7.5 km/h. V = 48(1) + 8(4) + (–2) = 78 m/s
What is the (a) magnitude of average velocity and (b) average
speed of the man, over the interval of time (i) 0 to 30 min. Example 17
(ii) 0 to 50 min and (iii) 0 to 40 min. (i) With the help of given graph find the instantaneous velocity
Sol.
at point F for the object whose motion the curve represents.
Time taken by man to go from his home to market,
(ii) Refer to fig. for the motion of an object along the x-axis,
Distance 2.5 1
t1 = = = h what is the instantaneous velocity of the object (a) at point
Speed 5 2
D? (b) at point C?
Time taken by man to go from market to his home, x, m
2.5 1 G
t2 = = h 15
7.5 3 D
1 1 5
\ Total time taken = t1 + t2 = + = h = 50 min. 10
2 3 6
® Displacement C F
(a) Average velocity V av = 5
Time E

Distance 0
(b) Average speed Vav = J 10 20 H t, s
Time Sol.
(i) Between 0 to 30 min (i) The tangent at F is the dashed line GH. Taking triangle GHJ,
® 2.5
V av = we have
= 5 km/h towards market
1/ 2 Dt = 24 – 4 = 20 s
2.5 Dx = 0 – 15 = – 15m
Vav = = 5 km/h Hence slope at F gives instantaneous velocity
1/ 2
(ii) Between 0 to 50 min Dx -15m
vF = = = – 0.75 m/s
Total distance travelled = 2.5 + 2.5 = 5 km. Dt 20 s
Total displacement = zero The negative sign tells us that the object is moving in the
® 5 –x direction.
V av = 0 \ Vav = = 6 km/h (ii) (a) Point D is a maximum of the x v/s t curve.
5/ 6
(iii) Between 0 to 40 min dx
Therefore, v = = 0.
Distance moved in 30 min (from home to market) dt
= 2.5 km. (b) Without the exact equation for x as function of t one
Distance moved in 10 min (from market to home) with cannot get a precise answer. The best we can do is to
draw the tangent line at point c and the slope in the
10
speed 7.5 km/h = 7.5 × = 1.25 km same way as in above problem. This will give the
60 answer
So displacement = 2.5 – 1.25 = 1.25 km (towards market) dx
vC =
Distance travelled = 2.5 + 1.25 = 3.75 km dt C » 1.3 m/s
® 1.25
V av = = 1.875 km/h. (towards market) Example 18
40 / 60 The graph of particle’s motion along the x-axis is given in fig.
3.75 Estimate the (a) average velocity for the interval from A to C;
Vav = = 5.625 km/h.
40 / 60 (b) instantaneous velocity at D.
40 Physics

Sol.
x (a) Average velocity for the interval from A to C,
(cm)
4 C
D ®
B 4.8 - 0
v= = 0.60 cm/s.
8.0 - 0
2
From the slope at each point
A
0 5 10 (b) At point D velocity v = – 0.48 cm/s.
t(s)

Speed
1. What is the numerical ratio of velocity to speed of an
object ?
(a) Always equal to one (c) t (d) All of the above
(b) Always less than one
9583963673
4. An athlete completes one round of a circular track of radius
(c) Always greater than one
R in 40 sec. What will be his displacement at the end of 2
(d) Either less than or equal to one. min. 20 sec ?
2. A car travels half the distance with constant velocity of (a) Zero (b) 2 R (c) 2pR (d) 7pR
40 kmph and the remaining half with a constant velocity of 5. The n - t graph for a particle is as shown. The distance
60 kmph. The average velocity of the car (in kmph is) travelled in the first four seconds is
(a) 40 (b) 45 (c) 48 (d) 50
(a) 12 m
3. Which of the following graph cannot possibly represent v 8
(m / s)
one dimensional motion of a particle? (b) 16 m
x 4
Velocity
(c) 20 m
t 0 2 4 6 t (sec)
(a) (b) t (d) 24 m

TOPIC 2.2 Acceleration, Kinematic Equations for Uniformly Accelerated Motion

ACCELERATION Variable or Non-Uniform Acceleration


Ø The rate of change of velocity of an object is called Ø An object is said to be moving with a variable acceleration
acceleration of the object. if its velocity changes by unequal amounts in equal
Ø Let v and v¢ be the velocity of the object at time t and t¢ intervals of time.
respectively, then acceleration of the body is given by Average Acceleration
® ® ®
Change in velocity Ø When an object is moving with a variable acceleration,
Acceleration ( a ) = = v '- v then the average acceleration of the object for the given
Time interval t '- t
(i) It is a vector quantity. motion is defined as the ratio of the total change in velocity
(ii) It is positive if the velocity is increasing and is of the object during motion to the total time taken.
negative if the velocity is decreasing. Total change in velocity
(iii) The negative acceleration is also called retardation i.e., Average acceleration =
Total time taken
or deceleration.
(iv) Unit : In S.I. system m/s2 v2
B
Velocity

In C.G.S. system cm/s2


(v) Dimensions : [M0LT–2] A
v1 C
Uniform Acceleration
Ø An object is said to be moving with a uniform acceleration O t1 t2
if its velocity changes by equal amounts in equal intervals
of time.
Motion in a Straight Line 41
®
Ø Suppose the velocity of a particle is v1 at time t1 and dv
® Ø Acceleration of particle as a function of x, a = v
dx
v 2 at time t2.
® ® ® NEET 2015
Then, Change in velocity = v 2 - v1 = Dv
Elapsed time in changing the velocity = t2 – t1 = Dt
® ® ® • Misconcept : If velocity is constant, then acceleration is
® v 2 - v1
Thus , a av = = Dv also constant.
t 2 - t1 Dt
• Concept : If there is no change in velocity or velocity is
® BC constant in magnitude and direction then the acceleration
Þ a av = = tan q = the slope of chord of v – t
AC is zero.
graph gives average acceleration. • Misconcept : If speed of object is constant, then
acceleration is zero.
• Concept : It is only true when object is moving in straight
Ø If a particle is speeding up, acceleration is in the direction line. When body moves in circular path with constant
of velocity. If its speed is decreasing, acceleration is in the
speed, then its direction changes at every point. So their
direction opposite to that of the velocity. This statement
is independent of the choice of the origin and the axis. acceleration is not zero, as velocity is not constant.
r
If any body is accelerated with acceleration a1 till time • Misconcept : If object moves with a high speed then its
r acceleration is also high.
t1 and acceleration a2 up to time t 2 then average
r a t +a t • Concept : High velocity does not mean high acceleration.
acceleration, aav = 1 1 2 2 Acceleration can be high, low or zero in high velocities. It
t1 + t 2
is only related to the change in velocity. If the change in
Instantaneous Acceleration
velocity is high, then the acceleration will be high.
Ø The acceleration of the object at a given instant of time or
at a given point of motion, is called its instantaneous • Misconcept : If acceleration is positive then the object
acceleration. speeds up.
• Concept : We should consider the sign of velocity while
talking about whether it is speeding up or slowing down.
B
v2 If the acceleration is positive and velocity is also positive
then we can say that object is speeding up in positive
A direction.
v1
C
t1 t2
Time
® ®
Ø Suppose the velocity of a particle at time t1 = t is v 1 = v (i) It is not essential that when velocity is zero acceleration
® ® ® must be zero. Ex. In vertical motion at the top point v =
and becomes v 2 = v + Dv at time t2 = t + Dt,
® 0 but a ¹ 0.i.e., a = g (acceleration due to gravity)
®
Then, a av = Dv
Dt
Ø If Dt approaches to zero then the rate of change of velocity
will be instantaneous acceleration. Instantaneous
acceleration
® ®
® Dv d v
a inst = Lim = NEET 2015
Dt ® 0 Dt dt Time
Ø Instantaneous acceleration at a point in the graph, shown
above, this point is given as dv
(ii) Velocity may vary but may be constant.
® dt
® dx æ ®ö ® (iii) The acceleration may vary but v may be constant, e.g.
v= or, ® d ç d x ÷ d 2 x
dt a= = 2 in uniform circular motion.
dt è dt ø dt (iv) If velocity decreases w.r.t. time then acceleration is called
Thus, instantaneous acceleration of an object is equal to
the second time derivative of the position of the object at retardation.
the given instant. Retardation a = tan(p – q) = –tanq
42 Physics

Example 19 dx d
Sol. v= = (at2 – bt3) = 2at – 3bt2
dt dt
An athlete takes 2 second to reach the maximum speed of
dv d
18 km/h from rest. What is the magnitude of his average Acceleration = = (2at – 3bt2) = 2a – 6bt
acceleration ? dt dt
Sol. Here, initial velocity u = 0, According to question, acceleration = 0
5 a
v = (vmax) = 18 km/h = 18 ´ = 5 m/s , t1 = 0, t2 = 2 sec. \ 2a – 6bt = 0 hence t =
18 3b
v-u Dv 5.0
aav = t - t = so, aav = = 2.5 m/s2 Example 24
2 1 Dt 2 If the displacement of a particle is (2t2 + t + 5)m then, what will
Example 20 be acc. at t = 5 sec?
A car moving with a velocity of 20 ms–1 is brought to rest in dx d
Sol. v = = (2t2 + t + 5) = 4t + 1 m/s
5 sec. by applying brakes. Calculate the retardation of the car. dt dt
v-u dv d
Sol. Here, u = 20 ms–1 , v = 0, t = 5 sec. Using a = , and a = = (4t + 1) Þ a = 4 m/s2
t dt dt
(0 - 20)
we have a = = – 4 m/s2 Example 25
5
–ve acceleration is known as retardation. Thus, retardation of The velocity of any particle is related with its displacement as;
the car = 4 ms–2. x = v + 1 , calculate acceleration at x = 5 m.
Example 21 Sol. x = v + 1 or, x 2 = v + 1 ; or, v = (x2 – 1)
The speed of a car as a function of time as shown in fig. Find the Therefore,
acceleration and distance travelled by the car in 8 second. dx
dv d 2
a= = (x - 1) = 2x - 0 = 2x v = 2x (x2 – 1)
dt dt dt
20 At x = 5 m, a = 2 × 5 (25 – 1) = 240 m/s2

10 KINEMATIC EQUATIONS FOR UNIFORMLY


ACCELERATED MOTION
2 4 6 8 Motion under uniform acceleration is described by the following
Time (s)
equations.
Sol. 1 a
v = u + at ; s = ut + at2; v2 = u2 + 2as and snth = u + (2n–1)
Dv 20 5 2 2
(i) Acceleration = = = = 2.5 m/s2 1. Velocity Time Relation
Dt 8 2
(ii) Distance travelled = area under speed–time graph dv
a= or dv = a dt
1 dt
= area of D = × 20 × 8 = 80 m If the velocity of particle at time t1 is v1 and at time t2 is v2
2
then,
Example 22 v2 t2
v t
The displacement of a particle is proportional to the cube of ò dv = ò a dt or [v]v 2 = a[t]t 2
1 1
elapsed time . How does the acceleration of the body depend on v1 t1
time elapsed ? v2 – v1 = a [t2 – t1] Þ v2 = v1 + a [t2 – t1]
Sol. Let x be the displacement at time t of an object in motion. If t1 = 0, v1 = u and t2 = t, v2 = v then,
Then according to question, x = kt3, where k is a constant. v = u + at
dx r r r
Velocity of object, v = = 3 kt2 (m/s) In vector form, v = u + at
dt
dv
2. Position Time Relation
Acceleration of object, a = = 3k ´ 2t = 6 k t. (m/s2) dx
dt By v = or dx = vdt
dt
i.e., a µ t. It means acceleration µ time. x2 t2 x2 t2
Þ ò dx = ò v dt Þ ò dx = ò (u + at) dt
Example 23 x1 t1 x1 t1
t2
The position x of a particle varies with time ‘t’ as x = at2 – bt3. x é 1 2ù
When will the acceleration of the particle become zero? Þ [x]x12 = ê ut + 2 at ú
ë û t1
Motion in a Straight Line 43

1 Ø Some Impossible Graphs


x2 – x1 = u (t2 – t1) + a (t 22 - t12 )
2
If t1 = 0, t2 = t, x1 = 0 and x2 = s then, Distance can never be negative At the same time a body cannot
be at two different positions
1
s = ut + at 2 +(ve)
2

Distance

Position
O
r r 1r2
In vector form, s = ut + at –(ve) Time
2
3. Velocity Position Relation t Time
O
dv dv dx dv
By a = = . =v
dt dt dx dx Speed of a body can never be negative
or a dx = v dv
v
x2 v2 é v2 ù 2
= ê 2 ú
x
ò a dx = ò v dv ; or,, a [x]x 2
1 ëê ûú v
x1 v1 1

Speed
or, 2 a(x2 – x1) = v 22 - v12 Time

If v1 = u; v2 = v and x2 – x1 = s then

v 2 = u 2 + 2as NEET 2020

In scalar product form : vr.vr = ur.ur + 2 ar.sr


• Misconcept : Equations of motion is always valid.
• Concept : Equations of motion is valid only when
Ø If a body is starting from rest and is moving with uniform acceleration is constant.
acceleration then distance travelled by the body in t second • Misconcept : Equations of motion depend on mass.
is proportional to t2 (i.e., s µ t2). • Concept : Equation of motions are independent of mass.
For example : the ratio of distance covered in 1 sec, 2 sec As if we release stone and feather in vacuum or, space
and 3 sec is 12 : 22 : 32 or 1 : 4 : 9. from same height, both will reach the earth together. It
4. Distance Travelled in nth Second of Uniformly does not happen on earth because of air resistance.
Accelerated Motion
1 1
Example 26
S th = Sn – Sn–1 = (u n + a n2) – [u(n–1) + a(n–1)2 The velocity acquired by a body moving with uniform
n 2 2
acceleration is 20 m/s in first 2 sec and 40 m/s in first 4 sec.
a Calculate its initial velocity.
So, S =u+ (2n - 1) NEET 2021
n th 2 v -v 40 - 20 20
Sol. a = 2 1 Þ a = = = 10 m/s2
All the kinematic equations discussed above hold good t 2 - t1 4-2 2
only for uniformly accelerated motion. v1 v2
u
They are not to be used in the case of variable acceleration.
Ø The Calculation of Velocity by Acceleration-time Graph:
Let a particle be moving with uniform acceleration t=0 t1 = 2 s t2 = 4 s
according to following a – t graph. Now, v1 = u + at1
v t2 Þ 20 = u + 10 × 2 Þ u = 0
D C
dv = a dt or, ò dv = ò a dt
Acc.
Example 27
u t1
A particle starts with a constant acceleration. At a time
v
or, (v)u = a (t) t 2
t t second speed is found to be 100 m/s and one second later
1 speed becomes 150 m/s. Find acceleration of the particle.
A B
\ v – u = a(t2 – t1) O Time
Sol. From first eqn of motion, v = u + at
Therefore difference in magnitude of velocity Þ 100 = 0 + at or 100 = at ....(1)
(v – u) = AB × AD Velocity after one second
v' = 0 + a(t + 1) Þ 150 = a(t + 1) .....(2)
or, v – u = area of rectangle ABCD = area under a – t On subtracting eqn.(1) from eqn. (2)
graph a = 50 m/s2
44 Physics

Example 28 Subtracting (ii) from (i),


0.3
A truck starts from rest with an acceleration of 1.5 m/s2 while 0.3 = 0 – 2a = – 2a or a=– = – 0.15 m/s2
2
a car 150 meter behind starts from rest with an acceleration of From (i), u = 1 – acc = 1 + 0.15 or u = 1.15 m/s
2 m/s2. (a) How long will it take before both the truck and car For the velocity of body at the end of 7th second,
are side by side and (b) How much distance is travelled by we have u = 1.15 m/s ; a = – 0.15 m/s2, v = ?, t = 7s
each? As, v = u + at = 1.15 + (– 0.15) × 7 = 0.1 m/s
Sol. Let car overtakes the truck after t second
Example 30
In time t distance travelled by truck
A person travelling at 43.2 km/h applies the brakes giving a
1 1
sT = at2 or sT = (1.5) t2 ....(1) deceleration of 6 m/s2 to his scooter. How far will it travel
2 2 before stopping ?
Truck
5
Car Sol. Here, u = 43.2 km/h = 43.2 ´ m/s ; 12 m/s
18
Deceleration ; a = 6 m/s2 ; v=0 ; s=?
As, v2 = u2 + 2as
150 m
Distance covered by car when car overtakes the truck 0 = (12)2 – 2 × 6 s
144
1 or, 144 = 2 × 6s or s= = 12 m
sc = (2) t2 [Q acc. of car = 2 m/s2] 12
2
1 Example 31
or (sT + 150) = (2) t2 ....(2)
2 A bullet going with speed 350 m/s enters in a concrete wall and
Dividing eqn. (2) by eqn. (1) penetrates a distance of 5 cm before coming to rest. Find the
deceleration.
sT + 150 2 150 20 4
= or 1+ = =
sT 1.5 sT 15 3 Sol. Here, u = 350 m/s, s = 5 cm, v = 0 m/s, a = ?

150 4 1 By using v2 = u2 + 2as 350 m/s 5m


or = -1 = or sT = 450 we get, 0 = u2 + 2as
sT 3 3
Distance travelled by truck ST = 450 m u2
or, u2 = –2as or, a =-
Distance travelled by car = 450 + 150 = 600 m 2s
Now by eqn (1) 350 ´ 350
1 2 1 or, a = - = –12.25 × 105 m/sec2
sT = at or 450 = × 1.5 × t2 2 ´ .05
2 2 Negative answer represents retardation.
450 ´ 2
t2 = Þ t = 300 ´ 2 = 24.5 sec. Example 32
1.5
Therefore car will overtake the truck after 24.5 sec. A passenger is standing d distance away from a bus. The bus
begins to move with constant acceleration a. To catch the bus,
Example 29 the passenger runs at a constant speed u towards the bus.
A body travels a distance of 2 m in 2 second and 2.2 m in next What must be the minimum speed of the passenger so that he
4 second. What will be the velocity of the body at the end of may catch the bus?
7th second from the start? Sol. Let the passenger catch the bus after time t.
Sol. Here, case (i) S = 2m and t = 2s
1 2
case (ii) S = 2 + 2.2 = 4.2 m and t = 2 + 4 = 6s The distance travelled by the bus, s1 = 0 + at ....(1)
Let u and a be the initial velocity and uniform acceleration of 2
the body. And the distance travelled by the passenger
From the 2nd equation of motion, s2 = ut + 0 ....(2)
1 Now the passenger will catch the bus if
S = ut + at2
2 d + s1 = s2 ....(3)
1
Case (i), 2 = (u × 2) + ( a × 22) Substituting the values of s1 and s2 from eqn. (1) and eqn. (2)
2 in eqn (3)
or 1 = u + a ....(i)
1 2
1 d+ at = ut
Case (ii), 4.2 = (u × 6) + ( a × 62) 2
2 1 [u ± u 2 - 2 ad ]
or 0.7 = u + 3a ....(ii) i.e., at2 – ut + d = 0 or t =
2 a
Motion in a Straight Line 45

So the passenger will catch the bus if t is real. 13a


or 20 = u + ...(i)
2
i.e., u2 ³ 2 ad or u ³ 2 ad a
So the minimum speed of passenger for catching the bus is Distance travelled in 9th second s9 = u + (2 × 9 – 1)
2
17
2 ad . or 24 = u + a ...(ii)
2
Example 33 Subtracting (ii) from (i), 4 = 2a or a = 2 m/s2
Putting this value of a in eqn (i)
A body travels a distance of 20 m in the 7th second and 24 m
in 9th second. How much distance shall it travel in the 15th 13
second? 20 = u + × 2 or 20 = u + 13
2
Sol. Here , s7 = 20 m ; s9 = 24 m, s15 = ?
or u = 20 – 13 = 7 m/s
Let u = initial velocity and a = uniform acceleration of the body.
a Distance travelled in 15th second
Distance travelled in nth second sn = u + (2n – 1)
2 a 2
a s15 = u + (2 × 15 – 1) = 7 + × 29 = 36 m
Distance travelled in 7th second s7 = u + (2 × 7 – 1) 2 2
2

6. The acceleration of a moving body can be found from


(a) v (b) v
(a) area under velocity-time graph
(b) area under distance-time graph (c) v2 (d) v 3
(c) slope of the velocity-time graph 9. The displacement x of a particle at the instant when its
(d) slope of distance-time graph velocity is v is given by v = 3x + 16 . Its acceleration
7. A particle starts its motion from rest under the action of a and initial velocity are
constant force. If the distance covered in first 10 seconds (a) 1.5 units, 4 units (b) 3 units, 4 units
is S1 and that covered in the first 20 seconds is S2, then: (b) 16 units, 1.6 units (d) 16 units, 3 units
(a) S2 = 3S1 (b) S2 = 4S1 10. A body starts from rest, what is the ratio of the distance
(c) S2 = S1 (d) S2 = 2S1
travelled by the body during the 4th and 3rd second ?
8. In a straight line motion, the distance covered is
proportional to the square root of the time taken. Then the 7 5 7 3
(a) (b) (c) (d)
acceleration of the particle is proportional to 5 7 3 7

TOPIC 2.3 Motion Under Gravity (Free Fall)

MOTION UNDER GRAVITY (FREE FALL) So if the body acquires velocity v after falling a distance h
in time t, equations of motion, viz.
Ø The most important example of motion in a straight line
with constant acceleration is motion under gravity. In case 1
of motion under gravity, v = u + at ; s = ut + at2 and v2 = u2 + 2as
2
(i) the acceleration is constant,
i.e., a = g = 9.8 m/s2 and directed vertically downwards. reduces to v = gt ....(1)
(ii) in vacuum, viscous force or thrust of the medium has
no effect on the motion. 1 2
h= gt ....(2) and
(1) Body falling freely under gravity : Taking initial position 2
as origin and downward direction of motion as positive, we v2 = 2gh ....(3) NEET 2013
have
u=0 [as body starts from rest] These equations can be used to solve most of the problems
a = +g [as acc. is in the direction of motion] of freely falling body.
46 Physics

If t is given If h is given If v is given +u


n n n n n n
use eq . (1) and eq (2) use eq . (2) and eq (3) use eq (3) and eq (1)
v
v = gt t = 2h t= g t = u/g t = 2u/g
g
O t
1 2 v 2
and h = and v = 2gh and h =
2 gt 2g

v a –u
h (B)
g If u is given, use eqn (3) and eqn (1)
tan =g u u2
t= ; h=
t t t g 2g
(i) If the body is dropped from a height H, as in time t, it

+ive
has fallen a distance h from its initial position, the
height of the body from the ground will be a O
t
1 2
h' = H – h with h = gt

–ive
2
1 2 g
(ii) As h = gt , i.e., h µ t2
2 –a (C)
So distance covered by a body fallen in time t, 2t, 3t etc.,
will be in the ratio of 12 : 22 : 32, i.e., square of integers.
(iii) The distance covered by a body fallen in the nth sec Ø A ball is thrown downwards from a building of height h
1 1 1 and it reaches after t seconds on earth. From the same
= h(n) – h(n – 1) = g(n)2 – g(n – 1)2 = g(2n – 1) building if two balls are thrown (one upwards and other
2 2 2 downwards) with the same velocity u and they reach the
So distances fallen in Ist, 2nd, 3rd sec etc. will be in the earth surface after t1 and t2 seconds respectively then
ratio of 1 : 3 : 5 i.e., odd integers only. NEET 2022
t= t1t2
(2) Body projected vertically up : Taking initial position as origin
and direction of motion (i.e., vertically up) as positive, IMPORTANT POINTS
here we have v = 0 [at highest point velocity = 0]
1. In case of motion under gravity for a given body, mass,
\ a = – g [as acc. is downwards while motion upwards] acceleration, and mechanical energy remain constant
If the body is projected with velocity u and reaches the while speed, velocity, momentum, kinetic energy and
highest point at a distance h above the ground in time t, the potential energy change.
equations of motion viz., 2. The motion is independent of the mass of the body, as
1 in any equation of motion, mass is not involved. This is
v = u + at, s = ut + at2 and v2 = u2 + 2as why a heavy and lighter body when released from the
2
1 same height, reach the ground simultaneously and with
reduces to 0 = u – gt, h = ut – gt2 and 0 = u2 – 2gh same velocity.
2
Substituting the value of u from first equation in second 2h
and rearranging these, i.e. t= and v = 2g h
g
u = gt ....(1) However, momentum, kinetic energy or potential energy
1 2 depend on the mass of the body (µ mass)
h = gt ....(2) 3. As from eqn.(2) time taken to reach a height h,
2
and 2
u = 2gh ....(3) 2h
tU =
These equations can be used to solve most of the problems g
of bodies projected vertically up. Similarly, time taken to fall down through a distance h,
If t is given, use eqn (1) and eqn (2) 2h
tD = 2h
1 so t U = t D =
u = gt and h = gt2 u2/2g g g
2 So in case of motion under gravity time taken to go up a
height h is equal to the time taken to fall down through
the same height h.
4. If a body is projected vertically up and it reaches a height
h
t = u/g t = 2u/g h, then u = 2gh and if a body falls freely through a
t
height h, then
(A)
If h is given, use eqn (2) and eqn (3) v = 2 gh = u
So in case of motion under gravity, the speed with which
2h a body is projected up is equal to the speed with which
t= ; u= 2g h
g it comes back to the point of projection.
Motion in a Straight Line 47

Some Important Graphs Related to Motion


All the following graphs are drawn for one-dimensional motion with uniform velocity or with constant acceleration.

Different case v-t graph s-t graph Important points


s
1. Uniform motion v (i) Slope of s-t graph
= v = constant
(ii) In s-t graph s = 0 at t = 0
v = constant
s = ut

t
t
v s
2. Uniformly accelerated (i) u = 0, i.e., v = 0 at t = 0
motion with u = 0 and (ii) u = 0, i.e., slope of s-t
s = 0 at t = 0 graph at t = 0, should
v = at
s = 12 at² be zero
(iii) a or slope of v-t graph
is constant
t t

3. Uniformly accelerated v s (i) u ¹ 0, i.e., v or slope of


motion with u ¹ 0 but s – t graph at t = 0 is not
s = 0 at t = 0 zero
v = u + at (ii) v or slope of s-t graph
1 at ² gradually goes on
s ut
2 increasing
t t
v s
4. Uniformly accelerated
motion with u ¹ 0 and s = s0 at t =0
s = s0 at t = 0
v = u + at
s s0 ut 1at ²
2
t t

v s
5. Uniformly retarded (i) Slope of s-t graph at
motion till velocity t = 0 gives u
becomes zero u s ut – 1 at² (ii) Slope of s-t graph at
v = u – at 2
t = t0 becomes zero
(iii) In this case u can’t be
t t zero
t0 t0

v (i) At time t = t0, v = 0 or


6. Uniformly retarded then s (Displacement) slope of s-t graph is
accelerated in opposite zero
u
direction (ii) In s-t graph slope or
velocity first decreases
then increases with
t opposite sign.
t0 t
t0
48 Physics

Motion Under Gravity in Presence of Air Resistance (b) Motion in downward direction : Total force during
An object is thrown with speed u in upward direction during its downward motion = mg – R
motion it experiences constant air resistance R in the direction
R
opposite to its motion. Hence, total acceleration, a = g -
(a) Motion in upward direction : Total force during upward m
motion = mg + R Time in downward motion (from IInd kinematic equation)
R 1æ Rö 2 u2 1æ Rö
Hence, total acceleration, a = g + h =0+ çg - ÷t Þ = g - ÷ t2
m 2è mø æ R ö 2 çè mø
2 çg + ÷
u è mø
Time in upward motion, t =
R u t upward
g+ Þ t=
m hence, <1
u 2 2 t downward
æRö
Maximum height, h = g2 - ç ÷
æ Rö èmø
2 çg + ÷
è mø

11. A stone released with zero velocity from the top of a tower, (a) u = 10 m / s, T = 2s (b) u = 10 m / s, T = 4s
reaches the ground in 4 sec. The height of the tower is (c) u = 20 m / s, T = 2s (d) u = 20 m / s, T = 4s
(g = 10 m/s2) 14. The water drops fall at regular intervals from a tap 5 m
(a) 20 m (b) 40 m (c) 80 m (d) 160 m above the ground. The third drop is leaving the tap at an
12. A ball dropped from a point A falls down vertically to C, instant when the first drop touches the ground. How far
through the midpoint B. The descending time from A to B above the ground is the second drop at that instant ? (Take
and that from A to C are in the ratio g = 10 m/s2)
(a) 1 : 1 (b) 1 : 2 (c) 1 : 3 (d) 1: 2 (a) 1.25 m (b) 2.50 m (c) 3.75 m (d) 5.00 m
13. A man throws a ball vertically upward and it rises through 15. If a ball is thrown vertically upwards with speed u, the
20 m and returns to his hands. The initial velocity (u) of the distance covered during the last t seconds of its ascent is
ball and for how much time (T) it remained in the air 1 2 1 2
respectively are (g = 10m/s2) (a) (u + gt)t (b) ut (c) gt (d) ut - gt
2 2

TOPIC 2.4 Relative Velocity in One Dimension

RELATIVE VELOCITY ¨ Relative velocity of a person w.r.t ground on the


Ø Relative velocity of an object A with respect to another moving escalator v = v1 + v2
object B, when both are in motion is the time rate at which
d d d 1 1 1 tt
object A changes its position with respect to object B. Þ = + Þ = + Þt= 1 2 NEET 2017
Ø Position of object A and B are given as t t1 t 2 t t1 t 2 t1 + t 2
xA = xOA + vAt and, xB = xOB + vBt Here t1 = time taken to travel up the stationary escalator,
xB – xA = (xOB – xOA) + (vB – vA) t t2 = time taken to travel up when person stands on the
or x = xO + (vB – vA) t moving escalator, t = time taken to walk up on the
x - xO moving escalator.
= vB – vA
t Special Cases
® ®
Ø If vA and vB be the respective velocities of object A (a) When the two objects move with equal velocities
and B, then relative velocity of A w.r.t. B is i.e., vA = vB or vB – vA = 0
® ® ®
v AB = v A – v B It means the two objects stay at constant distance apart
Similarly, relative velocity of B w.r.t. A is during the whole journey. In this case, the position-time
® ® ® graphs of two objects are parallel straight lines as shown in
v BA = v B – v A figure.
Motion in a Straight Line 49

For Q
x object B
(m) vB
x0B For
object A
O vA
x0A P

t (s)
–v B
(b) When the two objects move with unequal velocities (180– ) vAB
(i) When vA > vB, then vB – vA is negative. This shows
that the separation between two moving objects will
go on decreasing with time. After some time, the two Q' R
moving objects will meet and then the relative distance
between the objects will increase with time as shown The magnitude of the relative velocity vAB is given by
in figure.
vAB = v 2A + v B
2
+ 2v A v B cos (180° - q)
'
'A
t
jec

= v 2A + v B
2
- 2v A v B cos q
ob

Meeting '
t 'B
r

place jec
ste

r ob Ø Let a be the angle made by vAB with vA, then


Fa

we
Slo
x (m)

v B sin(180° - q) v B sin q
tan a = =
x0B vA + vB cos(180° - q) vA - vB cos q

x0A Time of meeting æ v B sin q ö


or a = tan–1 ç v - v cos q ÷
è A B ø
t(s)
®
(ii) When vB > vA , then vB – vA is positive. This shows Ða gives the direction of the relative velocity with v A .
that the separation between two moving objects will (i) When both the bodies are moving along parallel
go on increasing with time as shown in figure. straight lines in the same direction :
Then the angle between them is q = 0º
Faster object
(vB) vAB = v 2A + v B
2
- 2v A v B cos 0°
x
x0B = v 2A + v B
2
- 2v A v B [Q cos 0º = 1]
Slower object
x0A (vA) = (v A - v B ) 2 = (vA – vB)
Thus magnitude of relative velocity of A with respect
t to B is equal to the difference between the magnitude
(c) When two trains A and B move with same velocity v but in of individual velocities.
opposite direction (ii) When two bodies are moving along parallel straight
Ø The relative velocity of train A w.r.t. train B lines in the opposite direction i.e. q = 180º
® ® ®
v AB = v A – v B = v(i) ˆ - v( - ˆi) = 2v(i)
ˆ
Ø Relative velocity of train B w.r.t. A
\ vAB = v 2A + v B
2
- 2v A v B cos 180º
® ® ®
v BA = v B – vA = v(-ˆi) - v(i)
ˆ = 2v(-ˆi)
= v 2A + v B
2
+ 2v A v B [Q cos 180º = – 1]
Thus, when two trains cross each other in opposite
directions, then each train appears to move very fast (i.e.
double the actual speed) relative to the other. = (v A + v B )2 = (vA + vB)
(d) When the two bodies moving in directions inclined to each
other Thus magnitude of relative velocity of body A w.r.t.
Ø Relative velocity of A w.r.t B body B is equal to the sum of the magnitudes of
® ® ® vB Q individual velocities.
v AB = vA – vB
Ø The relative velocity of A B
with respect to B is given
by the diagonal OR of the When two bodies move in opposite directions, the
parallelogram OPRQ' as P magnitude of relative velocity of one with respect to the
shown in fig. O A vA other is equal to the sum of the magnitudes of two velocities.
50 Physics

Relative Velocity Due to Change of Frame 30 ´ 1000 m 25


Sol. Speed of police van = 30 km/h = = m/s
Ø Consider two frames of reference S and S'. 3600 s 3
S' 160
S Y' Speed of thief’s car = 192 km/h = m/s
Y
3
\ Relative speed of theif’s car w.r.t. police van
P
160 25
= - = 45 m/s
3 3
r Speed of bullet w.r.t. van = 150 m/s (Given)
r'
\ Speed with which bullet hits the car = 150 – 45 = 105 m/s
X'
O' Example 36
O X A swimmer can swim in still water at a rate of 4 km/h. If he
Suppose a particle P is observed from both the frames. swims in a river flowing at 3 km/h and keeps his direction (w.r.t.
The frames may be moving with respect to each other. water) perpendicular to the current. Find his velocity w.r.t. the
The position vector of the particle P w.r.t. the frame S is ground.
Sol. The velocity of the swimmer w.r.t. river water
® ® ®
r = OP . The position vector of the particle w.r.t. the frame vSR = 4.0 km/h in the direction perpendicular to the river. The
®
® ® velocity of river w.r.t. the ground is vRG = 3.0 km/h along the
S¢ is r ' = O´P . The position of the frame S¢ (the origin of length of river. Y
frame S¢) with respect to the frame S is OO¢.
® ®
The position vector of the O’ w.r.t. the frame S is rSS¢ = OO¢
VSG
From DOO¢P
® ® ®
VSR
OP = OO¢ + O¢ P
or ® ® ®
.....(1) X
r = r ¢ + rSS¢
VRG
The position of the particle with respect to S is equal to the ®
position of the particle with respect to S' plus the position The velocity of the swimmer w.r.t. the ground is vSG where
of S' with respect to S. ® ® ®
VSG = VSR + V RG
Ø If we differentiate eqn. (1) with respect to time, we get
2 2
d ® d ® d® VSG = VSR + VRG = 4 2 + 32
r = r' + rSS'
dt dt dt = 16 + 9 = 25 = 5 km / hr
or, ® ®
v =v + v
®
....(2) The angle q made with the direction of flow is
S S´ SS '
éV ù æ 4ö
®
= velocity of the particle with respect to S q = tan -1 ê SR ú = tan -1 ç ÷
vS V
ë RG û è 3ø
®
vS' = velocity of the particle with respect to S' and Relative Velocity of Rain w.r.t. the Moving Man
® ®
vSS' = velocity of the frame S' with respect to S. Ø A man walking west with velocity vm , represented by
®
OA . Let the rain be falling vertically downwards with
Example 34 ®
velocity ®
vr , represented by OB as shown in fig.
Two trains are moving eastward with velocities 10 ms–1 and
15 ms–1 on parallel tracks. Calculate the relative velocity of
slow train w.r.t. the fast train.
Sol. v1 = 10 ms–1 , v2 = 15 ms–1 A –vm
vm O C
Relative velocity of slow train w.r.t. the fast train W E

= v1 – v2 = 10 – 15 = – 5 ms–1
–ve sign shows that slow train appears to move westward w.r.t. vr v rm

fast train with velocity of 5 ms–1.


D
Example 35 B

A police van moving on a highway with a speed of 30 km/h fires


® ® ®
a bullet at thief’s car speeding away in the same direction with The relative velocity of rain w.r.t. man v rm = v r - v m ,
a speed of 192 km/h. If the muzzle speed of the bullet is ®
will be represented by diagonal OD of rectangle OBDC.
150 ms–1, with what speed does the bullet hit the thief’s car?
Motion in a Straight Line 51

\ `vrm = v 2r + v m
2
+ 2v r v m cos90º = v 2r + v 2m If the boy is running on the train in the direction opposite to the
motion of train, then
® ® ® ®
Ø If q is the angle which v rm makes with the vertical v = v rel. – v T
BD v æ vm ö (i) Suppose a train having length l is crossing a bridge of
direction then, tan q = = m or q = tan–1 ç
OB vr è v r ÷ø length L with constant speed v.
Here angle q is from vertical towards west and is written Then the time taken by the train to cross the bridge will be,
as q, west of vertical. l+L
t=
v

l L
In the above case if the man wants to protect himself from (ii) Suppose the train is accelerating with acceleration ‘a’ when
the rain, he should hold his umbrella in the direction of it crosses a bridge. It enters the bridge with speed u. Then
relative velocity of rain w.r.t. man i.e. the umbrella should be it will leave the bridge with speed v,
æ Thus; by using v2 = u2 + 2ax
-1 v m ö
hold making an angle q ç = tan west of vertical. v2 – u2 = 2ax
è v r ÷ø
v2 – u2 = 2a (l + L)
Example 37 or v = [u2 + 2a(l + L )]1/2.
A man is walking on a level road at a speed of 3 km/h. Raindrops River Problems
fall vertically with a speed of 4 km/h. Find the velocity of
Ø Minimum distance of approach
raindrops with respect to the man.
Sol. If we consider velocity of rain with respect to the man is C vr B
V km/h.
®
vrg = 4km/h Rain

vr
® vm
v d
vmg = 3km/h

Road
Relative velocity of man w.r.t. ground A
® ® ® Let d = width of river, vr = velocity of river water,
v mg = vm - v g ........(1) vm = velocity of swimmer
Velocity of rain w.r.t. ground Vmg = 3 km/hr The swimmer should swim in a direction such that resultant
® ® ®
v rg = v r - v g ........(2) ® ® ®
® ® ® Vrm v of vm and v r is along AB which is the shortest path.
Vrg= 4 km/hr
Velocity of rain w.r.t. man v rm = vr - v m vr d
On subtracting eqn (1) from eqn (2) sin q = v = vm 2 - v r 2 ; t =
vm ;
® ® ® –Vmg v m2 - vr 2
v rm = v rg - v mg
2
| vrm |= vrg 2 + vmg 2 = 42 + 32 = 5 km / h d = t vm
or, - v r2
3 Ø Minimum time of approach
or q = tan -1 æç ö÷
3
Direction : tan q =
4 è 4ø B C
IMPORTANT POINTS
If a body A is moving on a moving body B, the velocity of body vr vr
A relative to ground is the vector sum of the two velocities. d
®
vm
For example : If a boy is running with relative speed v rel on a v
®
train running with velocity vT relative to ground, the speed of A
®
the boy relative to ground v will be d
® ® ® Time of crossing the river t =
v = v rel. + vT vm cos q
If the boy is running on the train in the direction of motion of t = tmin when cos q = + 1 = cos 0° i.e. q = 0°
® ® ®
train, then v = v rel. + v d
T t min =
vm
52 Physics

Ø To cross the river in shortest time man should swim N


perpendicular to direction of flow.
Man will reach C instead of B /s
13 m

5 m/s
v x
If BC = x then, tan q = r =
vm d 12m/s
v E
so, x = r .d
vm The resultant velocity of woman = (12)2 + (5)2 = 13 m/s
Example 38 Let b be the angle made by the resultant velocity with the
A ship is streaming towards east at a speed of 12 ms–1. A direction of motion of the ship (i.e. east).
woman runs across the deck at a speed of 5 ms–1 in the direction 5 sin 90º 5
at right angle to the direction of motion of the ship i.e. towards \ tan b = = = 0.4167
north. What is the velocity of the woman relative to sea? 12 + 5 cos90° 12
Sol. The woman has two velocities simultaneously while \ b = 22º37' north of east.
running on the deck, one velocity is equal to the velocity of Thus, the direction of the velocity of the woman is 22º37¢ north
ship i.e. 12 m/s due east and other velocity is 5 m/s due north. of east.

16. A train of 150 metre length is going towards north direction


A A
at a speed of 10 m/s . A parrot flies at the speed of 5 m/s B

Position

Position
towards south direction parallel to the railway track. The
time taken by the parrot to cross the train is (c) (d) B
(a) 12 sec (b) 8 sec (c) 15 sec (d) 10 sec
17. Two trains are each 50 m long moving parallel towards Time Time
each other at speeds 10 m/s and 15 m/s respectively. After 19. A boat is sent across a river with a velocity of 8 km/hr. If
what time will they pass each other? the resultant velocity of boat is 10km/hr, then velocity of
2 the river is
(a) 5 sec (b) 4 sec (c) 2 sec (d) 6 sec
3 (a) 10 km/hr (b) 8 km/hr
18. Which one of the following represents the time-position (c) 6 km/hr (d) 4 km/hr
graph of two objects A and B moving with zero relative
speed?` 20. A boat moves with a speed of 5 km/h relative to water in a
river flowing with a speed of 3 km/h and a width of 1 km.
A The minimum time taken around a round trip is
Position

Position

A B (a) 5 minutes (b) 20 minutes


(a) B (b) (c) 30 minutes (d) 60 minutes
Time Time

Example 1
ds k k
A particle starts with initial velocity v0 and acceleration (ii) v= = v0 + t 2 Þ ds = (v0 + t 2 ) dt
dt 2 2
a = kt, where k is constant. Find (i) velocity and (ii)
displacement after time t. s t t
k 2
Þ ò ds = ò v0 dt + 2 ò t dt
dv
Sol. (i) Given : a = kt Þ = kt Þ dv = kt dt 0 0 0
dt
v t
kt 2 k t3 k
Þ ò dv = k ò t dt Þ v – v0 =
k
Þ v = v0 + t 2 Þ s = v0 t + Þ s = v0 t + t 3
2 2 2 3 6
v0 0

1 2 \ Displacement after time t = v 0 t + 1 kt 3


\ velocity v = v0 + kt 6
2
Motion in a Straight Line 53

Example 2 Sol. From the given graph x is positive upwards. Ball is dropped
from a height and its velocity increases in downward
A particle moves along x-axis with acceleration a = a0 direction due to gravity pull. In this condition v is negative
(1 – t/T) where a0 and T are constant. If velocity is zero at but acceleration of the ball is equal to acceleration due to
t = 0, then find the average velocity from t = 0 to the time gravity i.e., a = –g.
t when a = 0. When ball rebounds in upward direction its velocity is
dv æ tö positive but acceleration is a = –g
Sol. = a0 ç 1 - ÷ (a) The velocity-time graph of the ball is shown in fig (i).
dt è Tø
v t v
æ tö æ t2 ö
Þ ò dv = ò a0 ç1 - T ÷ dt Þ v = a0 çç t - 2T ÷÷
0 0
è ø è ø
t O t
dx æ t2 ö
Q = v so,
dt ò dx = ò v dt Þ x = ò 0 ç 2T ÷÷ dt
a ç t -
0 è ø
æ t2 t3 ö
Þ x = a0 çç - ÷ Qa=0Þ t=T Fig. (i)
÷ (b) The acceleration-time graph of the ball is shown in fig (ii).
è 2 6T ø
a
æ T 2 T3 ö
a0 ç -
ç 2 6T ÷÷ a T
displacement è ø= 0
Average velocity = = t
time T 3 O

Example 3 –g
A ball is projected vertically up with an initial speed of 20 m/
s on a planet where acceleration due to gravity is 10m/s2. Fig. (ii)
(a) How long does it take to reach the highest point? Example 5
(b) How high does it rise above the point of projection?
A body is released from a height and falls freely towards
(c) How long will it take for the ball to reach a point 10 m
the earth. Exactly 1 sec later another body is released. What
above the point of projection?
is the distance between the two bodies after 2 sec the release
Sol. As here motion is vertically upwards,
of the second body, if g = 9.8 m/s2?
\ a = – g and v = 0
Sol. The 2nd body falls for 2s, so
(a) From 1st equation of motion, i.e., v = u + at,
0 = 20 – 10t \ t = 2 sec. 1
(b) Using v2 = u2 + 2ax h2 = g(2)2 ...(1)
2
0 = (20)2 – 2 × 10 × h \ h = 20 m and 1st has fallen for 2 + 1 = 3 sec
1 2 1 1
(c) Using s = ut + at , 10 = 20t – × 10 × t2 h1 = g(3)2 ....(2)
2 2 2
\ Separation between two bodies after 2 sec of the release
t2 – 4t + 2 = 0 or t = 2 ± 2 , of 2nd body,
i.e. t = 0.59 sec. or 3.41 sec. 1
i.e., there are two times, at which the ball passes So, d = h1 – h2 = g(32 – 22) = 4.9 × 5 = 24.5 m
through 2
h = 10 m, while going up and then coming down. Example 6
Example 4 A stone is dropped into a well and the sound of impact
A ball is dropped and its displacement versus time graph is as of stone on the water is heard after 2.056 sec. of the release
of stone from the top. If acceleration due to gravity is 980
shown in given figure. (Displacement x is from ground and all
cm/sec2 and velocity of sound in air is 350 m/s, calculate
quantities are +ve upwards).
the depth of the well.
x Sol. If the depth of well is h and time taken by stone to reach the
bottom is t1, then
1
h= gt 2 ....(1)
2 1
Time taken by sound to reach surface
t h
t2 = ....(2)
(a) Plot qualitatively velocity versus time graph. 350
(b) Plot qualitatively acceleration versus time graph. But t1 + t2 = 2.056 ....(3)
54 Physics

Now as negative time is not physically acceptable, Here, u = 0, a = g


so t1 = 2 sec 1
\ Sn = g (2n – 1)
1 2
Depth of well, h = × 9.8 × 22 = 19.6 m Distance traversed in 1st second i.e., n = 1
2
1 1
Example 7 S1 = g (2 ´ 1 – 1) = g
2 2
If a body travels half of its total path in the last second of Distance traversed in 2nd second i.e., n = 2
its fall from rest, find : The time and height of its fall. Explain
the physically unacceptable solution of the quadratic time 1 3
S2 = g (2 ´ 2 – 1) = g
equation. (g = 9.8 m/s2) 2 2
1 2 Distance traversed in 3rd second i.e., n = 3
Sol. In time t, the body falls a height, h = gt
2 1 5
S3 = g (2 ´ 3 – 1) = g
[u = 0 as the body starts from rest] ....(1) 2 2
Now, as the distance covered in (t – 1) s is
1 3 5
1 \ S1 : S2 : S3 = g : g : g = 1: 3 : 5
h' = g(t – 1)2 ....(2) 2 2 2
2
From eqns (1) and (2) distance travelled in the last sec. Example 9
1 2 1 A person moves due east at speed 6 m/s and feels the wind
h – h' = gt – g(t – 1)2 is blowing to south at speed 6 m/s. (a) Find the actual velocity
2 2
of wind blow. (b) If person doubles his velocity then find
1 the relative velocity of wind blow w.r.t. man.
i.e., h – h' = g(2t – 1)
2 N
h
But according to given problem as (h – h' ) =
2
^
1 1 vm = 6i
i.e., h= g (2t – 1) Sol. (a) W E
2 2 45°

1 2 1 2
or gt = g (2t – 1) [as from eqn. (1) h = gt ]
2 2 vwm = – 6j
^
vw
or t2 – 4t + 2 = 0 S
r r r
-(-4) ± (-4)2 - 4 ´ 2 v wm = vw - vm
or t= =2± 2 r r r r
2 v w = vwm + vm = -6ˆj + 6iˆ ; v w = 6iˆ - 6jˆ
hence t = 0.59 s or t = 3.41 sec. r
0.59 s is physically unacceptable as it gives the total time t | vw | = 6 2 m/ s and it is blowing to S-E
taken by the body to reach ground lesser than one sec r
(b) Person doubles its velocity then v m = 12iˆ
while according to the given problem time of motion must
be greater than 1 s. N
1
Hence, t = 3.41 s and h = × (9.8) × (3.41)2 = 57 m
2
^
– 6i
Example 8
W 45° E
The ratio of distances traversed in successive intervals of
time when a body falls freely under gravity from certain
vwm
height is ^
– 6j
(a) l : 2 : 3 (b) l : 5 : 9 S
(c) 1 : 3 : 5 (d) 1: 2 : 3 But actual wind velocity remains unchanged.
nth r r r
Sol. (c) As we know, distance traversed in second v wm = v - v = (6iˆ - 6ˆj) - 12iˆ = -6iˆ - 6ˆj
w m
1
Sn = u + a (2 n – 1) Now relative velocity of wind is 6 2 m / s to S-W..
2
Motion in a Straight Line 55

Exercise 1 Home Assignment (NCERT Based MCQs)

Rest & Motion, Motion in One, Two, Three Dimensions, Frame of Reference,
TOPIC 2.1 Position, Path Length, Displacement, Speed and Velocity

1. Which of the following is a one dimensional motion ? 10. Which of the following is not possible for a body in
1. (a) Motion of snake Page-40 uniform motion? Page-41
(b) Motion of air particle

D isplacement
Displacement
(c) Motion of satellite
(d) Motion of train running on a straight track (a) (b)
2. The location of a particle has changed. What can we say
about the displacement and the distance covered by the Time Time
particle? Page-41 (c) Both (a) & (b) (d) None of these
(a) Neither can be zero (b) One may be zero 11. In 1.0 s, a particle goes from point A to point B, moving in
(c) Both may be zero (d) One is +ve, other is –ve a semicircle of radius 1.0 m (see Figure). The magnitude of
3. The displacement of a body is zero. The distance covered the average velocity is Page-42
(a) is zero Page-41 A
(b) is not zero (a) 3.14 m/s
(c) may or may not be zero
(d) depends upon the acceleration (b) 2.0 m/s 1.0m
4. The numerical ratio of displacement to distance for a
moving object is Page-41 (c) 1.0 m/s
(a) always less than 1 (b) always equal to 1
(c) always more than 1 (d) equal to or less than 1 (d) Zero
B
5. Which of the following can be zero, when a particle is in
motion for some time? Page-41 12. A body moves in straight line with velocity v1 for 1/3rd
(a) Distance (b) Displacement time and for remaining time with v2. Find average
(c) Speed (d) None of these velocity. Page-42
6. A person moved from A to B on a circular path as shown v1 2v 2 v1 v 2
in figure. If the distance travelled by him is 60 m, then the (a) + (b) +
3 3 3 3
magnitude of displacement would be : Page-41
2v1 v 2 2v 2
(Given cos135° = – 0.7) (c) + (d) v1 +
3 3 3
(a) 42 m 13. A particle moves in straight line with velocity 6 m/s and
(b) 47 m 3 m/s for time intervals which are in ratio 1:2 . Find
(c) 19 m average velocity. Page-42
(a) 2 m/s (b) 3 m/s
(d) 40 m
(c) 4 m/s (d) 5 m/s
7. Assertion : Displacement of a body may be zero when 14. A man leaves his house for a cycle ride. He comes back to
distance travelled by it is not zero. his house after half-an-hour after covering a distance of
Reason : The displacement is the longest distance one km. What is his average velocity for the ride?
between initial and final position. Page-40, 41 Page-42
(a) If both Assertion and Reason are correct and the (a) zero (b) 2 km h–1
Reason is a correct explanation of the Assertion. 1
(c) 10 km s–1 (d) km s -1
(b) If both Assertion and Reason are correct but Reason 2
is not a correct explanation of the Assertion. 15. A point traversed half of the distance with a velocity v0.
(c) If the Assertion is correct but Reason is incorrect. The half of remaining part of the distance was covered
(d) If both the Assertion and Reason are incorrect. with velocity v1 & second half of remaining part by v2
8. A particle moves 2m east then 4m north then 5 m west. velocity. The mean velocity of the point, averaged over
The distance is Page-40 the whole time of motion is Page-42
(a) 11 m (b) 10 m (c) –11 m (d) 5 m v 0 + v1 + v 2 2 v 0 + v1 + v 2
9. A particle moves from (2,3) m to (4,1) m. The magnitude (a) (b)
3 3
of displacement is Page-41 v 0 + .2 v1 + 2 v 2 2 v 0 (v1 + v 2 )
(c) (d)
(a) 2 m (b) 2 3 m (c) 2 2 m (d) 3 2m 3 (2v 0 + v1 + v 2 )
56 Physics

16. Statement I: Magnitude of average velocity is equal to 24. Choose the wrong statement from the following.
average speed. Page-47
Statement II: Magnitude of instantaneous velocity is not (a) The motion of an object along a straight line is a
rectilinear motion.
equal to instantaneous speed. Page-43
(b) The speed in general is less than the magnitude of
(a) Both statement I and II are correct. the velocity.
(b) Both statement I and II are incorrect. (c) The slope of the displacement-time graph gives the
(c) Statement I is correct but statement II is incorrect. velocity of the body.
(d) Statement II is correct but statement I is incorrect. (d) The area under the velocity-time graph gives the
displacement of the body.
17. Two buses P and Q start from a point at the same time and
25. Which of the following graph cannot possibly represent
move in a straight line and their positions are represented
one dimensional motion of a particle? Page-40
by XP(t) = at + bt2 and XQ(t) = ft – t2. At what time, both
x x
the buses have same velocity? Page-43

α- f α+ f a+ f f -a (a) t (b)
(a) (b) (c) (d) t
1+β 2(β – 1) 2(1 + b) 2(1 + b)

18. The fig given shows the time- speed


x
displacement curve of two P
Q
particles P and Q. Which of
the following statement is (c) (d) All of these
O t t
correct? Page-52

(a) Both P and Q move with uniform equal speed 26. The total distance travelled by the body in the given
(b) P is accelerated Q is retarded time is equal to Page-47

(c) Both P and Q move with uniform speeds but the speed (a) the area which v – t graph encloses with displacement axis
of P is more than the speed of Q (b) the area which x – t graph encloses with time axis
(d) Both P and Q move with uniform speeds but the speed (c) the area which v – t graph encloses with time axis
of Q is more than the speed of P. (d) the area which a – t graph encloses with axis
19. The distance travelled by a body is directly proportional 27. Choose the correct equation to determine distance in a
to the time taken. Its speed Page-42 straight line for a body with uniform motion.
(a) increases (b) decreases Page-47
(c) becomes zero (d) remains constant v
20. The slope of velocity-time graph for motion with uniform (a) s = (b) s = v2t
t
velocity is equal to Page-46, 47
1 2
(a) final velocity (b) initial velocity (c) s = ut + at (d) s = v × t2
2
(c) zero (d) none of these
28. The velocity time graph of the motion of the body is as
21. The ratio of the numerical values of the average velocity
shown below Page-47
and average speed of a body is Page-42
(a) unity (b) unity or less
A B
(c) unity or more (d) less than unity u
v(m/s)

22. The slope of the tangent drawn on position-time graph at


D E C
any instant is equal to the instantaneous
o t1 t2 t3 t (s)
(a) acceleration (b) force Page-43
(c) velocity (d) momentum The total distance travelled by the body during the
motion is equal to ____.
23. The displacement-time graphs of two particles A and B are
straight lines making angles of 30º and 60º respectively 1 1
(a) (AD + BE) × OC (b) (OA + BC) × OC
with the time axis. If the velocity of A is vA and that of B is 2 2
vB, the value of vA/vB is Page-41 1 1
(c) (OC + AB) × AD (d) (OA + AB) × BC
2 2
(a) 1/2 (b) 1 / 3 (c) 3 (d) 1/3
Motion in a Straight Line 57

TOPIC 2.2 Acceleration, Kinematic Equations for Uniformly Accelerated Motion

29. What is the rate of change of velocity of an object in 38. The dependence of velocity of a body with time is given
uniform motion ? Page-47 by the equation v = 20 + 0.lt2. The body is in
(a) Always equal to zero (a) uniform retardation Page-45
(b) Always less than one
(b) uniform acceleration
(c) Always greater than one
(d) Either less than or equal to one. (c) non-uniform acceleration
30. What determines the nature of the path followed by the (d) zero acceleration.
39. The deceleration experienced by a moving motorboat after
particle? Page-45
dv
(a) Speed (b) Velocity its engine is cut off, is given by = -KV3 where K is
(c) Acceleration (d) Both (b) and (c) dt
31. Acceleration of a particle changes when constant. If V0 is the magnitude of the velocity at cut-off,
(a) direction of velocity changes Page-45 the magnitude of the velocity at a time t after the cut-off is
(b) magnitude of velocity changes Page-45
V0
(c) speed changes (a) (b) V0e–Kt
(d) Both (a) and (b) (2V02Kt + 1)
32. The area under acceleration time graph gives (c) V0/2 (d) V0
Page-45 40. The displacement of a particle as a function of time is shown
(a) distance travelled (b) change in acceleration in figure. It indicates that Page-46
(c) force acting (d) change in velocity

Displacement in m
33. Which of the following is the correct expression of
instantaneous acceleration? Page-45
30
Dv dv 20
(a) a= (b) a =
( Dt)2 dt
10
2
d2 v æ Dv ö
(c) a = (d) a = ç ÷ 20 10
30 40 50
dt 2 è Dt ø Time in sec
34. The displacement of a particle is represented by the
following equation: S = 3t 3 + 7t 2 + 5t + 8 where 5 is in (a) the velocity of the particle is constant throughout
meter and t in second. The acceleration of the particle at (b) the acceleration of the particle is constant throughout
(c) the particle starts with a constant velocity and is
t = 15 is Page-45
accelerated
(a) 14 m/s2 (b) 18 m/s2 (c) 32 m/s2 (d) zero
(d) the motion is retarded and finally the particle stops
35. The velocity-time graph of a body is shown in fig. The
41. A particle moves along a straight line OX. At a time t (in
ratio of average acceleration during the intervals OA and
second) the distance x (in metre) of the particle from O is
AB is v (m/s)
Page-45
given by x = 40 + 12t – t3. How long would the particle
(a) 1
D C travel before coming to rest? Page-43
1 40 (a) 24 m (b) 40 m (c) 56 m (d) 16 m
(b)
2 42. A particle moves a distance x in time t according to equation
1 x = (t + 5)–1. The acceleration of particle is proportional to
(c)
3 Page-45
30º E 60º
(d) 3 O
A B t (s) (a) (velocity)3/2 (b) (distance)2
36. The distance time graph of a particle at time t makes angles (c) (distance)–2 (d) (velocity)2/3
45° with the time axis. After one second, it makes angle 60° 43. A particle is moving eastwards with a velocity of 5 ms–1. In
with the time axis. What is the acceleration of the particle? 10 seconds the velocity changes to 5 ms –1 northwards.
Page-46 The average acceleration in this time is Page-45

(a) 1 -2
3 - 1 (b) 3 + 1 (c) 3 (d) 1 (a) ms towards north
37. The displacement x of a particle along a straight line at 2
1
a1t a (b) ms - 2 towards north - east
time t is given by : x = a0 + + 2 t2. The acceleration of 2
2 3
1
the particle is Page-45 (c) ms - 2 towards north - west
a2 2a 2 a1 a2 2
(a) (b) (c) (d) a0 + (d) zero
3 3 2 3
58 Physics

44. It is given that t = px2 + qx, where x is displacement and t 52. Which of the following graphs gives the equation
is time. The acceleration of particle at origin is 1 2
x = vot + at Page-48
Page-45 2
2p 2q 2p 2q
(a) - (b) - (c) (d) v0
q 3
p 3
q 3
p3 x
45. An object, moving with a speed of 6.25 m/s, is decelerated (a) (b)
v
dv
at a rate given by: = -2.5 v where v is the instantaneous t t
dt
speed. The time taken by the object, to come to rest, would be
Page-45
(a) 2 s (b) 4 s (c) 8 s (d) 1 s v0
46. The position of a particle along the x-axis at certain times (c) v (d) None of these
is given below Page-45
t
t(s) 0 1 2 3 53. If a train travelling at 20 m/s is to be brought to rest in
x(m) –2 0 6 16 a distance of 200 m, then its retardation should be
Page-48
Which of the following describes the motion correctly?
(a) 1 m/s2 (b) 2 m/s2
(a) uniform acceleration
(c) 10 m/s2 (d) 20 m/s2
(b) uniform retardation
54. A body starts from rest and travels ‘s’ m in 2nd second,
(c) non-uniform acceleration
then acceleration is Page-47
(d) there is not enough data for generalization 2 2
47. The graph between displacement and time for a particle (a) 2s m/s (b) 3s m/s
moving with uniform acceleration is a/an 2 3
Page-48 (c) s m/s2 (d) s m/s2
3 2
(a) straight line with a positive slope 55. A bullet fired into a wooden block loses half of its velocity
(b) parabola
after penetrating 40 cm. It comes to rest after penetrating a
(c) ellipse
(d) straight line parallel to time axis further distance of Page-48

48. In a car race on straight road, car A takes a time t less than 22 40 20 22
(a) cm (b) cm (c) cm (d) cm
car B at the finish and passes finishing point with a speed 3 3 3 5
56. A body covers 26, 28, 30, 32 meters in 10th, 11th, 12th and 13th
'v' more than of car B. Both the cars start from rest and
seconds respectively. The body starts Page-48
travel with constant acceleration a1 and a2 respectively.
(a) from rest and moves with uniform velocity
Then 'v' is equal to: Page-48
(b) from rest and moves with uniform acceleration
2a1 a 2 (c) with an initial velocity and moves with uniform
(a) t (b) 2a1 a 2 t
a1 + a 2 acceleration
a1 + a 2 (d) with an initial velocity and moves with uniform velocity
(c) a1 a 2 t (d) t
2 57. A particle experiences constant acceleration for 20 seconds
49. Velocity time curve for a body projected vertically upwards is after starting from rest. If it travels a distance s1 in the first
Page-50 10 seconds and distance s2 in the next 10 seconds, then
(a) parabola (b) ellipse Page-48
(c) hyperbola (d) straight line (a) s2 = s1 (b) s2 = 2 s1
50. A bus starts moving with acceleration 2 m/s2. A cyclist 96 (c) s2 = 3 s1 (d) s2 = 4 s1
m behind the bus starts simultaneously towards the bus at
58. The distance travelled by a particle starting from rest and
20 m/s. After what time will he be able to overtake the bus?
4 –2
Page-48 moving with an acceleration ms , in the third second is:
3
(a) 4 sec (b) 8 sec
Page-48
(c) 18 sec (d) 16 sec
51. Stopping distance of a moving vehicle is directly 10 19
(a) 6 m (b) 4 m (c) m (d) m
proportional to Page-50 3 3
(a) square of the initial velocity 59. If a car at rest accelerates uniformly to a speed of 144 km/h in
(b) square of the initial acceleration 20 s, it covers a distance of Page-48
(c) the initial velocity
(d) the initial acceleration (a) 2880 m (b) 1440 m (c) 400 m (d) 20 m
Motion in a Straight Line 59

60. A bike accelerates from rest at a constant rate 5 m/s2 for 62. A car, starting from rest, accelerates at the rate f through a
some time after which it decelerates at a constant rate 3 m/s2 distance S, then continues at constant speed for time t and
to come to rest. If the total time elapsed is 8 second, the f
then decelerates at the rate to come to rest. If the total
maximum velocity acquired by the bike is given by 2
distance traversed is 15 S , then Page-48
Page-47 1 2
(a) 5 m/s (b) 10 m/s (c) 12 m/s (d) 15 m/s (a) S = ft (b) S = f t
6
61. A metro train starts from rest and in 5 s achieves 108 km/h. 1 2 1 2
(c) S = ft (d) S = ft
After that it moves with constant velocity and comes to 4 72
rest after travelling 45 m with uniform retardation. If total 63. A particle starting with certain initial velocity and uniform
acceleration covers a distance of 12 m in first 3 seconds
distance travelled is 395 m, find total time of travelling.
and a distance of 30 m in next 3 seconds. The initial velocity
Page-48 of the particle is Page-48
(a) 12.2 s (b) 15.3 s (c) 9 s (d) 17.2 s (a) 3 ms–1 (b) 2.5 ms–1 (c) 2 ms–1 (d) 1 ms–1

TOPIC 2.3 Motion Under Gravity (Free Fall)

64. A body is thrown vertically upwards. If air resistance is 71. The equation represented by the graph below is :
to be taken into account, then the time during which the Page-50
body rises is Page-49 1
(a) equal to the time of fall (a) y = gt
2
(b) less than the time of fall
(c) greater than the time of fall -1 t(s)
(b) y = gt O
(d) twice the time of fall 2
65. A body is thrown upwards and reaches half of its maximum y
1 2
height. At that position Page-49 (c) y = gt (m)
(a) its acceleration is minimum 2
(b) its velocity is maximum -1 2
(c) its velocity is zero (d) y = gt
(d) its acceleration is constant 2
66. Velocity-time curve for a body projected vertically 72. A body is projected vertically upwards. If t1 and t2 be the
upwards is Page-50 times at which it is at height h above the projection while
(a) parabola (b) ellipse ascending and descending respectively, then h is
(c) hyperbola (d) straight line Page-49
67. An object accelerated downward under the influence of
1
force of gravity. The motion of object is said to be (a) gt1t2 (b) gt1t2 (c) 2gt1t2 (d) 2hg
2
Page-49
(a) uniform motion 73. From a tower of height 400 m, a particle is thrown vertically
(b) free fall upwards with a speed of 10 m/s. If the time taken by it to
(c) non uniformly accelerated motion reach the highest point is T then the time taken by the
(d) None of these particle to hit the ground is Page-49
68. Free fall of an object (in vacuum) is a case of motion with (a) 20T (b) 15T
Page-49 (c) 10T (d) 5T
(a) uniform velocity (b) uniform acceleration 74. A rocket is fired upward from the earth’s surface such that
(c) variable acceleration (d) constant momentum it creates an acceleration of 19.6 ms–2. If after 5 s, its engine
69. A ball thrown vertically upwards after reaching a maximum
is switched off, the maximum height of the rocket from
height h, returns to the starting point after a time of l0 s. Its
displacement is Page-49
earth’s surface would be Page-49
(a) 980 m (b) 735 m (c) 490 m (d) 245 m
(a) h (b) 2 h (c) 10 h (d) zero
75. A man throws balls with same speed vertically upwards one
70. A ball is released from a height h. If t 1 and t2 be the time
after the other at an interval of 2 sec. What should be the speed
required to complete first half and second half of the
distance respectively. Then, choose the correct relation of throw so that more than two balls are in air at any time?
between t1 and t2. Page-49 Page-49
(a) Only with speed 19.6 m/s
(a) t1 = ( 2 ) t2 (b) t1 = ( )
2 - 1 t2 (b) More than 19.6 m/s

t 2 = ( 2 + 1) t1 t2 = ( 2 - 1) t1
(c) At least 9.8 m/s
(c) (d) (d) Any speed less then 19.6 m/s.
60 Physics

76. A ball is dropped from a high rise platform at t = 0 starting


æ 2h ö h
from rest. After 6 seconds another ball is thrown (a) T = 2h/v (b) T= ç ÷ +
downwards from the same platform with a speed v. The è g ø v
two balls meet at t = 18s. What is the value of v?(take g =
æ 2h ö h æ h ö 2h
10 m/s2) Page-49 (c) T= ç ÷+ (d) T = ç ÷÷ +
ç 2g
(a) 75 m/s (b) 55 m/s è v ø g è ø v
(c) 40 m/s (d) 60 m/s 82. A stone is dropped from the top of a building. When it
77. Match the Column I and Column II. Page-46, 47, 49 crosses a point 5 m below the top, another stone starts to
Column I Column II fall from a point 25 m below the top. Both stones reach the
(A) Cause increase in velocity (1) Linear motion
bottom of building simultaneously. The height of the
(B) Negative acceleration (2) Zero building is Page-49
(C) Motion exhibited by body moving (3) Distance (a) 35 m (b) 45 m (c) 25 m (d) 50 m
in a straight line 83. The balls are released from the top of a tower of height H at
(D) Area under a speed time graph (4) Acceleration regular interval of time. When first ball reaches at the
(E) Velocity of an upward throwing (5) Retardation th
æ n + 1ö
body at the peak point ground, the nth ball is to be just released and ç ÷ ball
(a) (A)®(4); (B)®(5); C®(1); (D)®(3) ; (E)®(2) è 2 ø
(b) (A)®(2); (B)®(1); C®(3); (D)®(4) ; (E)®(5) is at same distance ‘h’ from top of the tower. The value of h is
(c) (A)®(5); (B)®(2); C®(3); (D)®(1) ; (E)®(4) Page-49
(d) (A)®(2); (B)®(4); C®(1); (D)®(3) ; (E)®(5) 2 4 5H
3
78. From a building two balls A and B are thrown such that A is (a) H (b) H (c) H (d)
3 4 5 6
thrown upwards and B downwards (both vertically). If TA
84. A stone is dropped from a rising balloon at a height of 76 m
and TB are their respective time of flights then
above the ground and reaches the ground in 6s. What was
(a) TA > TB Page-49 the velocity of the balloon when the stone was dropped?
(b) TA = TB Take g = 10 m/s2. Page-49
(c) TA < TB
æ 52 ö æ 52 ö
(d) their time of flights depend on their masses. (a) ç ÷ m/s upward (b) ç ÷ m/s downward
è 3 ø è 3 ø
79. A ball is released from the top of tower of height h metre. It
takes T second to reach the ground. What is the position (c) 3 m/s (d) 9.8 m/s
in (m) from the ground of the ball in T/3 second ? 85. Let A, B, C, D be points on a vertical line such that AB =
Page-49 BC = CD. If a body is released from position A, the times of
h 7h 8h 17h descent through AB, BC and CD are in the ratio.
(a) (b) (c) (d) Page-49
9 9 9 18
80. A ball is dropped vertically from a height d above the (a) 1 : 3 - 2 : 3 + 2 (b) 1 : 2 - 1 : 3 - 2
ground. It hits the ground and bounces up vertically to a
height d/2. Neglecting subsequent motion and air (c) 1 : 2 - 1 : 3 (d) 1 : 2 : 3 - 1
resistance, its velocity v varies with the height h above the 86. Water drops fall at regular intervals from a tab which is
hm above the ground. After how many seconds does the
ground as Page-49
first drop reach the ground? Page-49

2h h h 2h
d (a) (b) (c) (d)
(a) h (b) g 2g 2g g
h
d 87. If two balls of masses m1 and m2(m1= 2m2) are dropped
from the same height, then the ratio of the time taken by
them to reach the ground will be Page-49
(a) m1 : m2 (b) 2m2 : m1
(c) 1 : 1 (d) 1 : 2
d d h 88. A boy standing at the top of a tower of 20m height drops a
(c) h (d) stone. Assuming g = 10 ms–2, the velocity with which it
hits the ground is Page-49
(a) 10.0 m/s (b) 20.0 m/s (c) 40.0 m/s (d) 5.0 m/s
81. A stone is dropped into a well in which the level of water is
89. What will be the ratio of the distances moved by a freely
h below the top of the well. If v is velocity of sound, the falling body from rest on 4th and 5th seconds of journey?
time T after which the splash is heard is given by Page-49
Page-49 (a) 4 : 5 (b) 7 : 9 (c) 16 : 25 (d) 1 : 1
Motion in a Straight Line 61

90. A ball released from a height falls 5 m in one second. In 4 æ 1 ö


seconds it falls through Page-49 (a) 2t (b) çè1 + ÷t

(a) 20 m (b) 1.25 m (c) 40 m (d) 80 m
91. From a balloon moving upwards with a velocity of 12 3t t
ms–1, a packet is released when it is at a height of 65 m from (c) (d)
2 2
the ground. The time taken by it to reach the ground is (g 97. The ratio of distances traversed in successive intervals of
= 10 ms–2) Page-49 time when a body falls freely under gravity from certain
(a) 5 s (b) 8 s (c) 4 s (d) 7 s height is Page-49
92. A ball is dropped from the top of a tower of height 100 m (a) l : 2 : 3 (b) l : 5 : 9
and at the same time another ball is projected vertically
upwards from ground with a velocity 25 ms –1. Then the (c) 1 : 3 : 5 (d) 1: 2 : 3
distance from the top of the tower, at which the two balls 98. A body dropped from top of a tower fall through 40 m
meet is Page-49 during the last two seconds of its fall. The height of tower
(a) 68.4 m (b) 48.4 m (c) 18.4 m (d) 78.4 m is (g = 10 m/s2) Page-49
93. A body released from the top of a tower falls through half (a) 60 m (b) 45 m (c) 80 m (d) 50 m
the height of the tower in 2 s. In what time shall the body 99. A stone thrown upward with a speed u from the top of the
fall through the height of the tower ? Page-49 tower reaches the ground with a velocity 3u. The height of
(a) 4 s (b) 3.26 s (c) 3.48 s (d) 2.828 s the tower is Page-49
94. Two bodies of masses m1 and m2 fall from heights h1 and (a) 3u2/g (b) 4u2/g (c) 6u2/g (d) 9u2/g
h2 respectively. The ratio of their velocities, when they hit 100. A stone thrown vertically upwards with a speed of 5 m/sec
the ground is Page-49 attains a height H1. Another stone thrown upwards from
the same point with a speed of 10 m/sec attains a height
h1 h1 m1h1 h12 H2. The correct relation between H1 and H2 is
(a) (b) (c) (d)
h2 h2 m1h 2 h 22 Page-49
95. A stone falls from a balloon that is descending at a uniform (a) H2 = 4H1 (b) H2 = 3H1
rate of 12 m/s. The displacement of the stone from the (c) H1 =2H2 (d) H1 = H2
point of release after 10 sec is Page-49 101. From a pole of height 10 m, a stone is thrown vertically
(a) 490 m (b) 510 m (c) 610 m (d) 725 m upwards with a speed 5 m/s. The time taken by the stone,
96. A body thrown vertically so as to reach its maximum to hit the ground, is n times that taken by it to reach the
height in t second. The total time from the time of highest point of its path. The value of n is
projection to reach a point at half of its maximum height [take g = 10 m/s2] Page-49
while returning (in sec) is Page-49 (a) 2 (b) 3 (c) 4 (d) 5

TOPIC 2.4 Relative Velocity in One Dimension

102. Two trains, each 40 m long are travelling in opposite (a) 60° west (b) 45° west
direction with equal velocity 20 m/s. The time of crossing (c) 30° west (d) 0°
is Page-51 105. A boat takes 2 hours to travel 8 km and back in still water
(a) 1s (b) 2s (c) 3s (d) Zero lake. With water velocity of 4 km h–1, the time taken for
going upstream of 8 km and coming back is
103. The graph shown below represent Page-52
Page-51
object B (a) 160 minutes (b) 80 minutes
(c) 100 minutes (d) 120 minutes
x02
Position

object A 106. A car is moving on a road and rain is falling vertically.


x01 Select the correct answer. Page-51
(a) The rain will strike the back screen only
Time (b) The rain will strike the front screen only
(a) A and B are moving with same velocity in opposite (c) The rain will strike both the screens
directions (d) The rain will not strike any of the screens
(b) velocity of B is more than A in same direction 107. If a boat can travel with a speed of v in still water, which of
(c) velocity of A is more than B in same direction the following trips will take the least amount of time?
Page-51
(d) velocity of A and B is equal in same direction
(a) travelling a distance of 2d in still water
104. The speed of a swimmer in still water is 16 m/s. The
(b) travelling a distance of 2d across (perpendicular to)
speed of river water is 8 m/s and is flowing due east. If
the current in a stream
he is standing on the south bank and wiches to cross
(c) travelling a distance d downstream and returning a
the river along the shortest path. The angle at which he
distance d upstream
should make his strokes w.r.t. north is given by
(d) travelling a distance d upstream and returning a
Page-52 distance d downstream
62 Physics
r with a speed of 10 km h–1. The time after which the distance
108. An object has velocity v1 relative to the ground.
r between them becomes shortest, is
An observer moving with a constant velocity v0 relative Page-51
to the ground measures the velocity of the object to be (a) 5 h (b) 5 2 h
r
v 2 (relative to the observer). The magnitudes of these (c) 10 2 h (d) 0 h
velocities are related by Page-51 110. A bus is moving with a velocity of 10 ms–1 on a straight
(a) v0 £ v1 + v 2 (b) v1 £ v 2 + v0 road. A scootorist wishes to overtake the bus in one minute.
If the bus is at a distance of 1.2 km ahead, then the velocity
(c) v2 £ v0 + v1 (d) All of these
with which he has to chase the bus is Page-51
109. A ship A is moving Westwards with a speed of 10 km h–1 (a) 20 ms–1 (b) 25 ms–1
and a ship B 100 km South of A, is moving Northwards (c) 60 ms–1 (d) 30 ms–1

1. The displacement-time graphs of two moving particles t1t2 t1t2


make angles of 30° and 45° with the x-axis as shown in the (a) t2 - t1 (b) t2 + t1
figure. The ratio of their respective velocity is:
NCERT | Page-44 | 2022 t1 + t2
(c) t1 – t2 (d)
2
6. Two cars P and Q start from a point at the same time in a
displacement

straight line and their positions are represented by xp(t) = at


+ bt2 and xQ (t) = ft – t2. At what time do the cars have the
same velocity? NCERT | Page-48 | Ph-II 2016

30° 45° a+f f -a a -f a+f


(a) ( ) (b) ( ) (c) (d)
0 time 2 l+b 2 l+b l+ b 2 ( b - 1)
(a) 1 : 1 (b) 1 : 2 (c) 1: 3 (d) 3 :1 7. 2
If the velocity of a particle is v = At + Bt , where A and B
2. The ratio of the distances travelled by a freely falling body are constants, then the distance travelled by it between
in the 1st, 2nd, 3rd and 4th second: NCERT | Page-50 | 2022 1s and 2s is : NCERT | Page-43 | 2016

(a) 1 : 4 : 9 : 16 (b) 1 : 3 : 5 : 7 3
(a) A + 4B (b) 3A + 7B
(c) 1 : 1 : 1 : 1 (d) 1 : 2 : 3 : 4 2
3. A small block slides down on a smooth inclined plane, 3 7 A B
(c) A+ B (d) +
starting from rest at time t = 0. Let Sn be the distance 2 3 2 3
travelled by the block in the interval t = n – 1 to t = n. 8. A particle of unit mass undergoes one-dimensional
S motion such that its velocity varies according to v(x) =
Then, the ratio n is NCERT | Page-48 | 2021 bx –2n
Sn +1
2n 2n - 1 2n - 1 2n + 1 where b and n are constants and x is the position of the
(a) (b) (c) (d) particle. The acceleration of the particle as the function of x,
2n - 1 2n 2n + 1 2n - 1
4. A ball is thrown vertically downward with a velocity of is given by: NCERT | Page-45 | 2015
20 m/s from the top of a tower. It hits the ground after (a) –2nb 2 x –4n–1 (b) –2b2 x –2n+1
some time with a velocity of 80 m/s. The height of the (c) –2nb 2 e –4n+1 (d) –2nb 2 x –2n–1
tower is : (g = 10 m/s2) NCERT | Page-49 | 2020
9. A stone falls freely under gravity. It covers distances h1,
(a) 340 m (b) 320 m (c) 300 m (d) 360 m h2 and h3 in the first 5 seconds, the next 5 seconds and
5. Preeti reached the metro station and found that the escalator the next 5 seconds respectively. The relation between
was not working. She walked up the stationary escalator in h1, h2 and h3 is NCERT | Page-49 | 2013
time t1. On other days, if she remains stationary on the
moving escalator, then the escalator takes her up in time t2. h2 h3
(a) h1 = = (b) h2 = 3h1 and h3 = 3h2
The time taken by her to walk up on the moving escalator 3 5
will be: (c) h1 = h2 = h3 (d) h1 = 2h2 = 3h3
NCERT | Page-52 | 2017
Motion in a Straight Line 63

1. If a car covers 2/5th of the total distance with v1 speed and Column I Column II
(A) u = + ve and a = + ve (1) v
3/5th distance with v2 speed then average speed is
1 v +v 2v1v2 5v1v2
(a) v1v2 (b) 1 2 (c) (d)
2 2 v1 + v2 3v1 + 2v2 t
0
2. The position x of a particle with respect to time t along x-
axis is given by x = 9t2 – t3 where x is in metre and t in (B) u = – ve, and a = + ve (2) v

second. What will be the position of this particle when it


achieves maximum speed along the +ve x direction?
(a) 54 m (b) 81 m (c) 24 m (d) 32 m t
0
3. A person starts his journey from centre ‘O’ of the park (C) u = + ve, and a = – ve (3) v
and comes back to the same position following path OPQO
as shown in the figure. The radius of path taken by the
person is 200 m and he takes 3 min 58 sec to complete his t
0
journey. The average speed of the person is (taken p = 3.14)
v
(D) u = – ve, and a = – ve (4)

t
0
(a) 1 ms–1 (b) 3 ms–1 (c) 5 ms–1 (d) 7 ms–1
(a) (A)®(3); (B)®(2); C®(1); (D)®(1)
4. Select the incorrect statements from the following.
I. Average velocity is path length divided by time (b) (A)®(2); (B)®(1); C®(4); (D)®(3)
interval (c) (A)®(1); (B)®(2); C®(3); (D)®(4)
II. In general, speed is greater than the magnitude of (d) (A)®(4); (B)®(3); C®(2); (D)®(1)
the velocity. 8. A particle starts from rest and travel a distance x with
III. A particle moving in a given direction with a non- uniform acceleration, then moves uniformly a distance 2x
zero velocity can have zero speed. and finally comes to rest after moving further 5x with
IV. The magnitude of average velocity is equal to the uniform retardation. The ratio of maximum speed to average
average speed speed is
(a) II and III (b) I and IV 5 5 7 7
(a) (b) (c) (d)
(c) I, III and IV (d) I, II, III and IV 2 3 4 5
5. A body starts from rest with an acceleration a1. After two 9. The graph shown in figure
seconds another body B starts from rest with an shows the velocity v versus
acceleration a2 . If they travel equal distances in fifth time t for a body.
second, after the starts of A, the ratio a1 : a2 will be equal Which of the graphs
to represents the corresponding
(a) 9 : 5 (b) 5 : 7 (c) 5 : 9 (d) 7 : 9 acceleration versus time
6. A car covers AB distance with first one–third at velocity graphs?
v1 ms–1, second one–third at v2 ms–1 and last one–third
at v3 ms–1. If v3 = 3v1, v2 = 2v1 and v1 = 11 ms–1 then the
average velocity of the car is. a a
(a) t (b) t

A v1 v2 v3 B

(a) 9 ms–1 (b) 15 ms–1 (c) 18 ms–1 (d) 25 ms–1


7. A particle is going along a straight line with constant
acceleration a, having initial velocity u. Then match the a a
columns : (c) t (d) t
64 Physics

10. Assertion : Displacement of a body is given by area 16. The speed verses time graph for a particle is shown in the
under velocity– time graph. figure. The distance travelled (in m) by the particle during
Reason : Displacement is a vector quantity. the time interval t = 0 to t = 5 s will be.
(a) If both Assertion and Reason are correct and the 10
Reason is a correct explanation of the Assertion. 8
u
–1 6
(b) If both Assertion and Reason are correct but Reason (ms )
is not a correct explanation of the Assertion. 4
(c) If the Assertion is correct but Reason is incorrect. 2
(d) If both the Assertion and Reason are incorrect. 1 2 3 4 5
11. A car is moving with speed of 150 km/h and after applying time
the break it will move 27 m before it stops. If the same car (s)
is moving with a speed of one third the reported speed (a) 5 (b) 10 (c) 15 (d) 20
then it will stop after travelling __________ m distance. 17. Column I Column II
(a) 3 (b) 7 (c) 9 (d) 11 (A) Zero acceleration (1) Retardation
12. A goods train accelerating uniformly on a straight railway (B) Positive slope of velocity (2) Speed
track, approaches an electric pole standing on the side of time graph
track. Its engine passes the pole with velocity u and the (C) Speed in a direction (3) Constant motion
guard’s room passes with velocity v. The middle wagon of (D) Acts in opposite direction of (4) Acceleration
the train passes the pole with a velocity motion
u+v 1 2 (E) Slope of a distance time graph (5) Velocity
(a) (b) u + v2 (a) (A)®(4); (B)®(5); C®(1); (D)®(3) ; (E)®(3)
2 2
(b) (A)®(2); (B)®(1); C®(3); (D)®(4) ; (E)®(5)
æ u 2 + v2 ö (c) (A)®(5); (B)®(2); C®(3); (D)®(1) ; (E)®(4)
(c) uv (d) ç ÷ (d) (A)®(3); (B)®(4); C®(5); (D)®(1) ; (E)®(2)
è 2 ø
18. Two bodies P and Q are moving along positive x-axis their
13. A particle is moving with constant acceleration ‘a’. r
position-time graph is shown below. If VPQ is velocity of
Following graph shows n2 versus x(displacement) plot. The r
acceleration of the particle is P w.r.t. Q and VQP is velocity of Q w.r.t P then
r r x
(a) | VPQ | = | VQP | = constant P
r Q
(b) VPQ towards origin
r
(c) VQP towards origin
(d) both (a) and (c) t

19. The distance x covered by a particle in one dimensional


motion varies with time t as x2 = at2 + 2bt + c. If the
acceleration of the particle depends on x as x n, where n is
an integer, the value of n is.
(a) 1 (b) 3 (c) 5 (d) 7
20. A body is thrown vertically upwards with a velocity u.
(a) 1 m/s2 (b) 3 m/s2 (c) 5 m/s2 (d) 9 m/s2 Select the incorrect statements from the following.
14. Select the correct statements from the following I. Both velocity and acceleration are zero at its
I. A body can have constant velocity but variable highest point
speed II. Velocity is maximum and acceleration is zero at the
II. A body can have constant speed but variable highest point.
velocity III. Velocity is maximum and acceleration is g
III. A body can have zero velocity but non–zero downwards at its highest point.
acceleration (a) I and II (b) II and III
(a) I and II (b) II and III (c) I and III (d) I, II and III
(c) I and III (d) I, II and III 21. A particle is moving with speed v = b x along positive
15. If the velocity of a body related to displacement x is given x-axis. Calculate the speed of the particle at time t = t (assume
by u = 5000 + 24x m/s , then the acceleration of the that the particle is at origin at t = 0).

body is. b2 t b2 t b2 t
(a) (b) (c) 2
b t (d)
(a) 8 m/s2 (b) 12 m/s2 (c) 16 m/s2 (d) 20 m/s2 4 2 2
Motion in a Straight Line 65

22. All the graphs below are intended to represent the same A B
v (m/s) 4
motion. One of them does it incorrectly. Pick it up. 2
velocity
distance S D t (in s)
0
1 2 3 4 5 6
(a) position (b) time –2
C
37 49
velocity (a) m (b) 12 m (c) 11 m (d) m
position 3 4
29. The position of a particle as a function of time t, is given
(c) time (d) time by x(t) = at + bt2 – ct3 where, a, b and c are constants.
When the particle attains zero acceleration, then its velocity
23. Match the Column I and Column II. will be:
Column I Column II b2 b2 b2 b2
(a) a+ (b) a + (c) a + (d) a +
(A) Displacement (1) Slope of x – t graph 4c 3c c 2c
(B) Velocity (2) Slope of tangent to 30. Assertion : Position-time graph of a stationary object is
x – t Curve a straight line parallel to time axis.
(C) Acceleration (3) Area under v – t curve Reason : For a stationary object, position does not
(D) Instantaneous (4) Slope of v – t graph change with time.
(a) If both Assertion and Reason are correct and the
velocity (5) Area under x – t curve Reason is a correct explanation of the Assertion.
(a) (A)®(4); (B)®(2); C®(1); (D)®(3) (b) If both Assertion and Reason are correct but Reason
(b) (A)®(2); (B)®(4); C®(3); (D)®(1) is not a correct explanation of the Assertion.
(c) (A)®(3); (B)®(1); C®(4); (D)®(2) (c) If the Assertion is correct but Reason is incorrect.
(d) If both the Assertion and Reason are incorrect.
(d) (A)®(2); (B)®(4); C®(1); (D)®(3)
31. A ball is thrown vertically upwards with a velocity of 19.6
24. The velocity of the bullet becomes one third after it ms–1 from the top of a tower. The ball strikes the ground
penetrates 4 cm in a wooden block. Assuming that bullet after 6s. The height from the gound up to which the ball
is facing a constant resistance during its motion in the
block. The bullet stops completely after travelling at æ kö
(4 + x) cm inside the block. The value of x is: can rise will be çè ÷ø m. The value of k is (use g = 9.8 m/s2)
5
(a) 2.0 (b) 1.0 (c) 0. 5 (d) 1.5 (a) 250 (b) 392 (c) 497 (d) 682
25. A small toy starts moving from the position of rest under a 32. Train A and train B are running on parallel tracks in the
constant acceleration. If it travels a distance of 10 m in t s, opposite directions with speeds of 36 km/hour and 72 km/
the distance travelled by the toy in the next t s will be : hour, respectively. A person is walking in train A in the
(a) 10 m (b) 20 m (c) 30 m (d) 40 m direction opposite to its motion with a speed of 1.8 km/
26. Assertion : The relative velocity between any two hour w.r.t train A. Speed (in ms–1) of this person as observed
bodies moving in opposite direction is equal to sum of from train B will be close to : (take the distance between the
the velocities of two bodies. tracks as negligible)
Reason : Sometimes relative velocity between two bodies (a) 29.5 ms–1 (b) 28.5 ms–1
(c) 31.5 ms –1 (d) 30.5 ms–1
is equal to difference in velocities of the two.
(a) If both Assertion and Reason are correct and the 33. Statement I: A body may be accelerated even when it is
Reason is a correct explanation of the Assertion. moving uniformly.
Statement II: When direction of motion of the body is
(b) If both Assertion and Reason are correct but Reason
changing, the body must have acceleration.
is not a correct explanation of the Assertion.
(a) Both statement I and II are correct.
(c) If the Assertion is correct but Reason is incorrect. (b) Both statement I and II are incorrect.
(d) If both the Assertion and Reason are incorrect. (c) Statement I is correct but statement II is incorrect.
27. A scooter accelerates from rest for time t1 at constant rate (d) Statement II is correct but statement I is incorrect.
a1 and then retards at constant rate a2 for time t2 and 34. A passenger train of length 60 m travels at a speed of 80 km/
t1 hr. Another freight train of length 120 m travels at a speed
comes to rest. The correct value of t will be of 30 km/h. The ratio of times taken by the passenger train
2
a1 + a2 a1 a2 a +a to completely cross the freight train when: (i) they are
(a) (b) (c) (d) 1 2 moving in same direction, and (ii) in the opposite directions
a1 a2 a1 a2
is:
28. The velocity (v) and time (t) graph of a body in a straight
11 5 3 25
line motion is shown in the figure. The point S is at 4.333 (a) (b) (c) (d)
seconds. The total distance covered by the body in 6 s is: 5 2 2 11
66 Physics

35. A ball is thrown up vertically with a certain velocity so 40. Statement I: The equation of motion can be applied only
that, it reaches a maximum height h. Find the ratio of the if acceleration is along the direction of velocity and is
h constant.
times in which it is at height while going up and coming
3 Statement II: If the acceleration of a body is zero then
down respectively. its motion is known as uniform motion.
2 -1 3+ 2 (a) Both statement I and II are correct.
(a) (b) (b) Both statement I and II are incorrect.
2 +1 3- 2
(c) Statement I is correct but statement II is incorrect.
3 -1 1 (d) Statement II is correct but statement I is incorrect.
(c) (d) 41. Water droplets are coming from an open tap at a particular
3 +1 3
rate. The spacing between a droplet observed at 4th sec-
36. A ball of mass 0.5 kg is dropped from the height of 10m. ond after its fall to the next droplet is 34.3 m. At what rate
The height, at which the magnitude of velocity becomes the droplets are coming from the tap ?
equal to the magnitude of acceleration due to gravity, is
(Use g = 10 m/s2). (Take g = 9.8 m/s2)
(a) 5 m (b) 10 m (c) 15 m (d) 20 m (a) 3 drops / 2 seconds (b) 2 drops / second
37. A juggler throws balls vertically upwards with same initial (c) 1 drop / second (d) 1 drop / 7 seconds
velocity in air. When the first ball reaches its highest 42. Water drops are falling from a nozzle of a shower onto the
position, he throws the next ball. Assuming the juggler floor, from a height of 9.8 m. The drops fall at a regular interval
throws n balls per second, the maximum height the balls of time. When the first drop strikes the floor, at that instant,
can reach is the third drop begins to fall. Locate the position of second
(a) g/2n (b) g/n (c) 2gn (d) g/2n2 drop from the floor when the first drop strikes the floor.
38. The position, velocity and acceleration of a particle moving (a) 4.18 m (b) 2.94 m (c) 2.45 m (d) 7.35 m
with a constant acceleration can be represented by: 43. Statement I: A body is momentarily at rest when it reverses
its direction of motion.
a(t) Statement II: A body cannot have acceleration if its velocity
acceleration

v ( t)
position

velocity

(a) x(t )
is zero at a given instant of time.
(a) Both statement I and II are correct.
t t t (b) Both statement I and II are incorrect.
(c) Statement I is correct but statement II is incorrect.
a(t)
acceleration

x(t) (d) Statement II is correct but statement I is incorrect.


position

velocity

v (t )
(b) 44. A balloon was moving upwards with a uniform velocity of
10 m/s. An object of finite mass is dropped from the balloon
t t t
when it was at a height of 75 m from the ground level. The
height of the balloon from the ground when object strikes
a(t)
acceleration

x(t) the ground was around:


position

velocity

(c) v(t)
(takes the value of g as 10 m/s2)
(a) 300 m (b) 200 m (c) 125 m (d) 250 m
t t t
45. A helicopter rises from rest on the ground vertically
v( t) a(t) upwards with a constant acceleration g. A food packet is
acceleration

x(t)
position

velocity

dropped from the helicopter when it is at a height h. The


(d)
time taken by the packet to reach the ground is close to [g
is the accelertion due to gravity] :
t t t
39. Two balls A and B are placed at the top of 180 m tall tower. 2 æhö h
Ball A is rele ased from the top at t = 0 s. Ball B is thrown (a) t = ç ÷ (b) t = 1.8
vertically down with an initial velocity 'u' at t = 2 s. After a 3 ègø g
certain time, both balls meet 100 m above the ground. Find
the value of 'u' in ms–1. [use g = 10 ms–2] : 2h æhö
(c) t = (d) t = 3.4 ç ÷
(a) 10 (b) 15 (c) 20 (d) 30 3g ègø
Motion in a Straight Line 67

1. A hunter tries to hunt a monkey with a small, very poisonous 6. Starting from rest a particle moves in a straight line with
arrow, blown from a pipe with initial speed v0. The monkey is 3p
hanging on a branch of a tree at height H above the ground. acceleration a = (25 – t2)1/2 m/s2 for 0 £ t £ 5s, a = m/s2
8
The hunter is at a distance L from the bottom of the tree. The for t > 5s. The velocity of particle at t = 7s is:
monkey sees the arrow leaving the blow pipe and immediately
Page-45
loses the grip on the tree, falling freely down with zero initial
(a) 11 m/s (b) 22 m/s (c) 33 m/s (d) 44 m/s
velocity. The minimum initial speed v0 of the arrow for hunter
7. Column I Column II
to succeed while monkey is in air is Page-49, 50
(A) Distance travelled (1) zero acceleration
g ( H 2 + L2 ) gH 2 by a body
(a) (b) 1
2H H 2 + L2 (B) Uniform velocity (2) ut + at2
2
g H 2 + L2 2gH 2 (C) Speedometer (3) instantaneous speed
(c) (d) u2
H H 2 + L2 (D) Height of a vertically (4)
2. Among the four graph shown in the figure there is only 2g
one graph for which average velocity over the time interval thrown body Page-43, 48, 49

(O, T) can vanish for a suitably chosen T. Which one is it? (a) (A)®(2, 3); (B)®(2); C®(3, 4); (D)®(1, 5)
Page-46 (b) (A)®(1, 2); (B)®(3); C®(5); (D)®(4)
x x (c) (A)®(1, 5); (B)®(1); C®(3); (D)®(4, 5)
(d) (A)®(2); (B)®(4); C®(1); (D)®(3)
8. The displacement ‘x’ (in meter) of a particle of mass ‘m’ (in
(a) t (b) kg) moving in one dimension under the action of a force,
t is related to time ‘t’ (in sec) by t = x + 3 . The displacement
x x
of the particle when its velocity is zero, will be
Page-43
(c) (d) (a) 2 m (b) 4 m (c) 0 m (d) 6 m
t t 9. The distance travelled by a body moving along a line in
3. A particle when thrown, moves such that it passes from time t is proportional to t3. The acceleration-time (a, t)
same height at 2 and 10 seconds, then this height h is : graph for the motion of the body will be
Page-49, 50 Page-43, 45

(a) 5g (b) g (c) 8g (d) 10g


4. Which graph corresponds to an object moving with a a
a
constant negative acceleration and a positive velocity? (a) (b)
Page-46 t
t

(a) Velocity (b) Velocity a a


(c) (d)
Time Time
t t
10. A ball is dropped from a height of 5 m onto a sandy floor
(c) Velocity (d) Velocity and penetrates the sand upto 10 cm before coming to rest.
Find the retardation of the ball in sand assuming it to be
Distance Distance uniform. Page-49, 50
5. A lift is coming from 8th floor and is just about to reach (a) 490 m/s2 (b) 610 m/s2
4th floor. Taking ground floor as origin and positive
direction upwards for all quantities, which one of the (c) 720 m/s2 (d) 810 m/s2
following is correct? Page-45 11. A bird flies with a speed of 10 km/h and a car moves
(a) x < 0, v < 0, a > 0 (b) x > 0, v < 0, a < 0 with uniform speed of 8 km/h. Both start from B towards
(c) x > 0, v < 0, a > 0 (d) x > 0, v > 0, a < 0 A (BA = 40km) at the same instant. The bird having reached
68 Physics

A, flies back immediately to meet the approaching car. As 16. Statement I: A body falling freely may do so with
soon as it reaches the car, it flies back to A. The bird repeats constant velocity.
this till both the car and the bird reach A simultaneously. Statement II: The body falls freely, when acceleration of
The total distance flown by the bird is Page-42 a body is not equal to acceleration due to gravity.
(a) 80 km (b) 40 km (a) Both statement I and II are correct.
(c) 50 km (d) 30 k m (b) Both statement I and II are incorrect.
12. The displacement of a particle is given by (c) Statement I is correct but statement II is incorrect.
y = a + b t + c t2 – d t4 (d) Statement II is correct but statement I is incorrect.
The initial velocity and acceleration are respectively 17. The motion of a particle along a straight line is described
Page-43, 45 by equation :
(a) b, – 4 d (b) – b, 2 c x = 8 + 12t – t3
(c) b, 2 c (d) 2 c, – 4 d where x is in metre and t in second. The retardation of the
particle when its velocity becomes zero, is :
13. A particle starts from origin O from rest and moves with a
Page-45
uniform acceleration along the positive x-axis. Identify all
(a) 24 ms–2 (b) zero (c) 6 ms–2 (d) 12 ms–2
figures that correctly represents the motion qualitatively
18. A particle moving along x-axis has acceleration f, at time
(a = acceleration, v = velocity, x = displacement, t = time)
Page-42, 46 æ tö
t, given by f = f 0 çè 1 - ÷ø , where f0 and T are constants.
T
The particle at t = 0 has zero velocity. In the time interval
between t = 0 and the instant when f = 0, the particle’s
velocity (vx) is Page-46
(A) (B) v
1 1
(a) f T2 (b) f T2 (c) f0T (d) f0T
2 0 2 0
19. The displacement x of a particle varies with time t as x =
ae-at + bebt, where a, b, a and b are positive constants. The
velocity of the particle will Page-43
(C) (D) (a) be independent of a and b
(b) drop to zero when a = b
(c) go on decreasing with time
(a) (B), (C) (b) (A) (d) go on increasing with time
(c) (A), (B), (C) (d) (A), (B), (D) 20. The displacement of a particle varies with time (t) as: s =
at2 – bt3. The acceleration of the particle will be zero at
14. Assertion: A particle starting from rest and moving with
uniform acceleration travels' a length of x and 3x in first time t equal to Page-45

two and next two-seconds. a a 3b 2a


Reason: Displacement is directly proportional to velocity. (a) (b) (c) (d)
b 3b a 3b
Page-47 21. Three different objects of masses m1, m2 and m3 are allowed
(a) If both Assertion and Reason are correct and the to fall from rest and from the same point O along three
Reason is a correct explanation of the Assertion. different frictionless paths. The speeds of the three
(b) If both Assertion and Reason are correct but Reason objects on reaching the ground will be in the ratio of
is not a correct explanation of the Assertion. Page-49
(c) If the Assertion is correct but Reason is incorrect. (a) m1 : m2 : m3 (b) m1 : 2m2 : 3m3
(d) If the Assertion is incorrect and Reason is correct.
1 1 1
15. Two fixed points A and B are 20 metres apart. At time t = 0, (c) 1 : 1 : 1 (d) : :
the distance between a third point C and A is 20 meters and m1 m2 m3
the distance between C and B is 10 metres. The component 22. The displacement of a particle is given by x = (t – 2)2 where
of velocity of point C along both CA and CB at any instant x is in metre and t in second. The distance covered by the
is 5m/s. Then the distance between A and C at the instant particle in first 4 seconds is Page-43, 47
all the three points are collinear will be Page-43 (a) 4 m (b) 8 m
(a) 5 m (b) 15 m (c) 10 m (d) 25 m (c) 12 m (d) 16 m
Motion in a Straight Line 69

23. A ball is dropped from a high rise platform at t = 0 starting reaches the floor, the distance of the man above the floor
from rest. After 6 seconds another ball is thrown will be: Page-49
downwards from the same platform with a speed v. The (a) 9.9 m (b) 10.1 m(c) 10 m (d) 20 m
two balls meet at t = 18s. What is the value of v?
25. Two bodies, A (of mass 1 kg) and B (of mass 3 kg), are
(take g = 10 m/s2) Page-49
dropped from heights of 16m and 25m, respectively. The
(a) 75 m/s (b) 55 m/s (c) 40 m/s (d) 60 m/s
ratio of the time taken by them to reach the ground is
24. A man of 50 kg mass is standing in a gravity free space at
Page-49
a height of 10 m above the floor. He throws a stone of 0.5
kg mass downwards with a speed 2 m/s. When the stone (a) 12/5 (b) 5/12 (c) 4/5 (d) 5/4

ANSWER KEYS
CDQs 2.1 Classroom Discussion Questions
1 (d) 2 (c) 3 (d) 4 (b) 5 (b)
CDQs 2.2 Classroom Discussion Questions
6 (c) 7 (b) 8 (d) 9 (a) 10 (a)
CDQs 2.3 Classroom Discussion Questions
11 (c) 12 (d) 13 (d) 14 (c) 15 (c)
CDQs 2.4 Classroom Discussion Questions
16 (d) 17 (b) 18 (b) 19 (c) 20 (c)
Exercise 1 Home Assignment (NCERT Based MCQs)
1 (d) 12 (a) 23 (d) 34 (c) 45 (a) 56 (c) 67 (b) 78 (a) 89 (b) 100 (a)
2 (a) 13 (c) 24 (b) 35 (c) 46 (a) 57 (c) 68 (b) 79 (c) 90 (d) 101 (c)
3 (c) 14 (a) 25 (d) 36 (a) 47 (b) 58 (c) 69 (d) 80 (a) 91 (a) 102 (a)
4 (d) 15 (d) 26 (c) 37 (b) 48 (c) 59 (c) 70 (d) 81 (b) 92 (d) 103 (d)
5 (b) 16 (d) 27 (c) 38 (c) 49 (d) 60 (d) 71 (d) 82 (b) 93 (d) 104 (c)
6 (b) 17 (d) 28 (c) 39 (d) 50 (b) 61 (d) 72 (a) 83 (b) 94 (b) 105 (a)
7 (c) 18 (c) 29 (a) 40 (d) 51 (a) 62 (d) 73 (c) 84 (a) 95 (c) 106 (b)
8 (a) 19 (d) 30 (d) 41 (c) 52 (c) 63 (d) 74 (b) 85 (b) 96 (b) 107 (a)
9 (c) 20 (c) 31 (c) 42 (a) 53 (a) 64 (b) 75 (b) 86 (a) 97 (c) 108 (d)
10 (c) 21 (b) 32 (d) 43 (c) 54 (c) 65 (d) 76 (a) 87 (c) 98 (b) 109 (a)
11 (b) 22 (c) 33 (b) 44 (a) 55 (b) 66 (d) 77 (a) 88 (b) 99 (b) 110 (d)
Exercise 2 NEET Past Year MCQs
1 (c) 2 (b) 3 (c) 4 (c) 5 (b) 6 (b) 7 (c) 8 (a) 9 (a)
Exercise 3 Multi-Concept Exercise
1 (d) 6 (c) 11 (a) 16 (d) 21 (b) 26 (b) 31 (b) 36 (a) 41 (c)
2 (a) 7 (b) 12 (d) 17 (d) 22 (b) 27 (c) 32 (a) 37 (d) 42 (d)
3 (b) 8 (c) 13 (a) 18 (d) 23 (c) 28 (a) 33 (a) 38 (a) 43 (c)
4 (c) 9 (b) 14 (b) 19 (b) 24 (c) 29 (b) 34 (a) 39 (d) 44 (c)
5 (c) 10 (b) 15 (b) 20 (d) 25 (c) 30 (a) 35 (b) 40 (d) 45 (c)
Exercise 4 Master Stroke
1 (a) 4 (c) 7 (c) 10 (a) 13 (d) 16 (d) 19 (d) 22 (b) 25 (c)
2 (b) 5 (a) 8 (c) 11 (c) 14 (c) 17 (d) 20 (b) 23 (a)
3 (d) 6 (b) 9 (b) 12 (c) 15 (b) 18 (c) 21 (c) 24 (b)

You might also like